You are on page 1of 131

MCQs in Pathology

Vinay Kumar, MBBS, MD, FRCPath


Alice Hogge and Arthur Baer
Distinguished Service Professor,
Chairman Emeritus
Department of Pathology

!
Correct answer: A. Integrins are involved in firm attachment of neutrophils to endothelium at the
site of infection or tissue injury, a critical step in recruitment of leukocytes from the circulation
into inflamed tissues.
B. Migration of leukocytes after they leave blood vessels is mediated mainly by chemokines.
Integrins do contribute to adhesion of cells to extracellular matrix, but migration is not
greatly impacted by deficiency of integrins.
C, D, E. These reactions are the consequence of leukocyte activation and integrins are not
involved in these. They occur after the cells have migrated to the site of infection.

!
Correct answer: A. Selectins and their ligands are expressed on leukocytes and activated
endothelial cells, and mediate low-affinity interactions that form and are broken by the
shear force of flowing blood. Because of these interactions, the leukocytes roll along
endothelium at sites of inflammation. This is the first step in the passage of leukocytes
from the circulation into tissues, and it gives the leukocytes the chance to initiate the
second step of firm attachment (which is mediated by integrins).
B. Integrins are involved in firm adhesion of leukocytes to endothelium
C. Leukotriene B4 is a chemotactic agent
D. NADPH oxidase catalyzes the generation of reactive oxygen species, part of the
respiratory burst
E. Defensins are anti-microbial peptides stored in leukocyte granules.
!
Correct answer: C. Upon activation, neutrophils produce reactive oxygen species (ROS) such
as superoxide in granules, which are converted to the more potent microbicidal halides by
the action of the granule enzyme myeloperoxidase. These halides (and other free radicals)
destroy phagocytosed microbes.
A. Degranulation is triggered by calcium influx and cytoskeletal activation in response to
stimuli that activate neutrophils.
B. Prostaglandins are produced by phospholipase A2-mediated digestion of membrane
phospholipids to generate arachidonic acid, which are then converted to prostaglandins by
the action of cyclooxygenase.
D. Oxygen consumption is increased in activated neutrophils, not decreased.
E. Hydrogen peroxide is also derived from the ROS superoxide but this process does not
require myeloperoxidase.
!
Correct answer: A. Macrophages remove noxious agents such as microbes and the debris of
damaged cells. They also produce growth factors that promote angiogenesis and
deposition of collagen, the two key processes in tissue repair (healing). Different subsets
of macrophages may be primarily involved in destroying microbes and clearing damaged
tissues and in promoting repair.
B. Plasma cells are antibody secreting cells that play important roles in defense against
microbes but not in tissue repair.
C. Neutrophils are important for destroying ingested microbes and dead material, but are
minor sources of growth factors that contribute to tissue repair.
D. The physiologic function of eosinophils is mainly to destroy helminthic parasites. They
also contribute to tissue injury in allergic reactions.
E. Epithelioid cells are a type of activated macrophages with abundant eosinophilic
cytoplasm. They are found typically in granulomas, in which macrophage activation is a
prominent process.
!
Correct answer: B. This lesion is typical of granulomatous inflammation, a type of chronic
inflammation that is dominated by activated macrophages. In most cases, the macrophages
are activated by CD4+ T cells, which produce the cytokine interferon-γ, the most potent
macrophage-activating cytokine.
A. The complement system is involved in innate immune reactions and in antibody- and
immune complex-mediated reactions, not in T cell- and macrophage-dependent
granulomatous inflammation.
C. Bradykinin is a peptide that causes vascular dilation.
D. Nitric oxide is a microbicidal substance produced in macrophages in inflammation and
host defense. It is also a vasodilator.
E. Prostaglandins are products of arachidonic acid that have numerous actions on blood
vessels and leukocytes in acute inflammation, but are not potent macrophage activators.

Question #1
A 24-year-old woman gives birth to a term infant after an uncomplicated pregnancy. Three
days after birth, the infant vomits all oral feedings. On physical examination, the
abdomen is distended and tender, and bowel sounds are reduced. An abdominal
ultrasound scan shows marked colonic dilation above a narrow segment in the distal
sigmoid region. A biopsy specimen from the narrowed region shows an absence of
ganglion cells in the muscle wall and submucosa. Which of the following is most likely
to produce these findings?
A Colonic atresia
B Hirschsprung disease
C Intussusception
D Necrotizing enterocolitis
E Volvulus

Correct answer: B. Colonic obstruction in a newborn indicates a congenital disease. The


absence of ganglion cells in the wall of the colon is diagnostic for Hirchsprung
disease.
A, colonic atresia is very rare. It may also present with obstruction, but the biopsy
would not show an absence of ganglion cells
C, Intussusception is telescoping of the bowel through a distal portion, sometimes
caused by a tumor but usually without an identifiable cause
D, necrotizing enterocolitis is a bacterial infection that presents with bloody diarrhea. It
is a complication of prematurity. It typically involves terminal ileum, cecum, and
right colon; it does not present with obstruction
E, Volvulus (twisting of the intestine) also presents later in life and may be diagnosed by
ultrasound or radiologically.

Question #8

A 53-year-old woman with no prior illnesses has a routine checkup by her physician. On

examination she has a blood pressure of 150/95 mm Hg. If her hypertension remains

untreated for years, which of the following cellular alterations would most likely be seen

in her myocardium?

A Apoptosis

B Fatty change
C Hyperplasia

D Hypertrophy

E Metaplasia

Correct answer: D. In response to the increased work load for the heart to pump against

high arterial pressure, the myocardial cells will undergo compensatory hypertrophy

(increase in cell size), enabling them to function despite the greater stress.

A, apoptosis may occur in late stages if the hypertrophied myocardium cannot cope with

the stress, but this is not the initial response to hypertension

B, fatty change represents a form of reversible cell injury. It is also the consequence of

abnormal lipid metabolism and is seen most often in hepatocytes (where lipids are

conjugated to proteins for excretion)

C, hyperplasia (increase in cell numbers) is a form of adaptation which occurs in response

to growth factors and other stimuli in tissues capable of cell proliferation (which the

myocardium is not)

E, metaplasia is a change of one adult cell type to another; it is often in response to a

chronic irritant
Question #9

A 22-year-old woman becomes pregnant. A fetal ultrasound examination at 13 weeks’

gestation shows her uterus measures 7 × 4 × 3 cm. At delivery of a term infant, her uterus

measures 34 × 18 × 12 cm. Which of the following cellular processes has contributed

most to the increase in her uterine size?

A Endometrial glandular hyperplasia

B Myometrial fibroblast proliferation

C Endometrial stromal hypertrophy

D Myometrial smooth muscle hypertrophy

E Vascular endothelial hyperplasia

Correct answer: D. Myometrial cells undergo hypertrophy (increase in size) in response to

hormonal stimuli, in order to accommodate the growing fetus.

A, although endometrial gland hyperplasia (increased number of cells) does occur in

response to hormones, which function as growth factors, it would not increase the size of

the uterus

B, fibroblasts are a minor component of myometrium and have limited proliferative

capacity.

C, endometrial stromal cells are also a minor component of the uterine wall and thus do

not contribute much to wall thickness

E, vascular endothelial cells do not undergo much proliferation (hyperplasia) and do not

respond to hormones produced during pregnancy


Question #10

A 20-year-old woman breastfeeds her infant. On examination, her breasts are slightly

increased in size. Milk can be expressed from both nipples. Which of the following

processes that occurred in her breasts during pregnancy enables her to breastfeed the

infant?

A Ductal metaplasia

B Epithelial dysplasia

C Intracellular lipid deposition

D Hormone induced cellular proliferation

E Stromal hypertrophy

Correct answer: D. Proliferation of mammary gland epithelial cells induced by prolactin,

and accompanied by increased production of milk proteins, is essential for lactation

A, metaplasia (change in phenotype of ductal lining cells) does not occur during lactation

B, dysplasia is a premalignant change often in response to external stresses (such as in

bronchial epithelium in smokers)

C, lipids are synthesized and secreted in milk, not deposited in epithelial cells

E, hypertrophy of the breast stroma does occur during pregnancy, contributing to the

increased size, but is not the process that allows lactation


Question #11

A 16-year-old boy sustained blunt trauma to his abdomen when he struck a bridge abutment

at high speed while driving a motor vehicle. Peritoneal lavage shows a hemoperitoneum,

and at laparotomy, a small portion of the left lobe of the injured liver is removed. Two

months later, a CT scan of the abdomen shows that the liver has nearly regained its size

before the injury. Which of the following processes best explains this CT scan finding?

A Apoptosis

B Dysplasia

C Hyperplasia

D Hydropic change

E Steatosis

Correct answer: C. Proliferation of liver cells in response to loss of tissue (called

compensatory hyperplasia) accounts for restoration of liver size. Liver cells have great

capacity to divide.

A, apoptosis leads to cell death and shrinkage of an organ, not restoration

B, dysplasia is a premalignant change often in response to external stresses (such as in

bronchial epithelium in smokers)

D, hydropic change is fluid accumulation, sometimes an indicator of early cell injury

E, steatosis is lipid deposition, which occurs in the liver in conditions of abnormal lipid

metabolism
Question #12

A 71-year-old man has had difficulty with urination, including hesitancy and increased

frequency, for the past 5 years. A digital rectal examination reveals that his prostate gland

is palpably enlarged to twice normal size. A transurethral resection of the prostate is

performed, and the microscopic appearance of the prostate “chips” obtained is that of

nodules of glands with intervening stroma, without morphologic atypia. Which of the

following pathologic processes has most likely occurred in his prostate?

A Apoptosis

B Dysplasia

C Fatty change

D Hyperplasia

E Hypertrophy

Correct answer: D. In response to androgens, the prostate undergoes enlargement because

of hyperplasia (proliferation) of stromal cells and glandular elements and not

hypertrophy ( E)

A, apoptosis leads to cell death and shrinkage of an organ

B, dysplasia is a premalignant change often in response to external stresses (such as in

bronchial epithelium in smokers)

C, fatty change (steatosis) is lipid deposition, which occurs in some forms of reversible cell

injury and in the liver in conditions of abnormal lipid metabolism


18 A 32-year-old man is involved in a vehicular accident and sustains fractures of the right

femur and tibia and the left humerus. The fractures are stabilized surgically. He is in

stable condition for 2 days, but then suddenly becomes severely dyspneic. Which of the

following complications from his injuries is the most likely cause of his sudden

respiratory difficulty?

A Cardiac rupture

B Fat embolism

C Pulmonary edema

D Pulmonary infection

E Bleeding into the pleural space

Answer, B Fat embolism is a likely cause in this setting following trauma to multiple

bones.

A Cardiac rupture is most commonly a complication of recent myocardial infarction, for

which there is no evidence. When caused by severe chest trauma it would occur

shortly after the injury.

C Pulmonary edema is most commonly cause by heart failure, for which there is no

evidence.

D Pulmonary infection would not be expected to present with sudden onset of severe

dyspnea.

E Bleeding into the pleural space would be an unlikely cause of dyspnea in this setting

except as an immediate complication of acute injury.


19 A 22-year-old woman with an uncomplicated pregnancy develops sudden dyspnea with

cyanosis and hypotension intrapartum during routine vaginal delivery of a term infant.

She has a generalized seizure and becomes comatose. Her condition does not improve

over the next 2 days. Which of the following findings is most likely to be present in her

peripheral pulmonary arteries?

A Aggregates of platelets

B Amniotic fluid

C Fat globules

D Gas bubbles

E Thromboemboli

Answer, B Amniotic fluid embolism is a rare but severe and frequently fatal

complication of childbirth.

A Aggregates of platelets may cause symptoms related to vessel obstruction, most often

in hemolytic uremic syndrome (HUS), thrombotic thrombocytopenia purpura (TTP),

and heparin-induced thrombocytopenia, but none of these are acute complications of

childbirth.

C Fat globules can cause similar symptoms if they reach the vasculature, but this

typically occurs following trauma to bones.

D Gas bubbles in blood vessels may occur in decompression sickness or may enter the

circulation as a complication of instrumentation that inadvertently introduces gas.

E Thromboemboli arise in the setting of deep vein thrombosis of leg veins. In this case

amniotic fluid emboli are far more likely


20 A 31-year-old man is on a scuba diving trip and descends to a depth of 50 m in the Blue

Hole off the coast of Belize. After 30 minutes, he has a malfunction in his equipment and

quickly returns to the boat on the surface. He develops difficulty breathing within 5

minutes, with dyspnea and substernal chest pain, followed by a severe headache and

vertigo. An hour later, he develops severe, painful myalgias and arthralgias. These

symptoms abate within 24 hours. Which of the following occluding his arterioles is the

most likely cause of his findings?

A Fat globules

B Fibrin clots

C Nitrogen gas bubbles

D Platelet thrombi

E Ruptured atheromatous plaque

Answer, C Nitrogen gas bubbles. The story is classic for “the bends” (decompression

sickness), in which rapid decompression cause nitrogen to come out of solution and

form bubbles in vessels and other sites such as joints.

A Fat globules (fat emboli) in the blood are a complication of physical trauma.

B Fibrin clots may be a complication of disseminated intravascular coagulation (DIC),

which is not associated with rapid decompression.

E Ruptured atheromatous plaque also is not a complication of decompression.


21 A 53-year-old man with congestive heart failure develops pulmonary Streptococcus

pneumoniae infection after a bout of influenza. After recuperating for 2 weeks, he notes

pleuritic chest pain. The pain is caused by the development of the lesion shown in the

figure (Fig 4-17A, BP10). Which of the following events has most likely occurred in this

man?

A Acute pulmonary venous congestion

B Chronic pulmonary venous congestion

C Pulmonary edema

D Pulmonary infarction and hemorrhage

E Pulmonary venous thrombosis

Answer, D Pulmonary infarction and hemorrhage. The presence of a “red” infarct is

consistent with a pulmonary embolism, which produced infarction and subsequent

hemorrhage in this patient in whom oxygenation was compromised by another

superimposed process (congestive heart failure, pneumonia)

A Acute pulmonary venous congestion would produce a diffuse red lung, not a focal

area as seen in this image.

B Chronic pulmonary venous congestion might produce a diffusely brownish lung, due

to recurrent small hemorrhages and deposition of hemosiderin.

C Pulmonary edema would be associated with heavy, wet lungs

E Pulmonary venous thrombosis is very rare and may produce changes associated with

pulmonary hypertension, rather than pulmonary infarction.


22 An 80-year-old woman with dysuria for 1 week now has a fever. On examination, her

temperature is 37.9° C, pulse 103/min, and blood pressure 80/40 mm Hg. She has right

flank pain. A urinalysis shows numerous WBCs. Her plasma lactate is increased. Urine

and blood culture grow Escherichia coli. This clinical condition is most likely initiated by

which of the following processes?

A Activation of innate immune cells and endothelium by microbial products

B Anaerobic glycolysis resulting in lactic acidosis

C Disseminated intravascular coagulation resulting in widespread hypoxia

D Triggering of mast cells resulting in release of vasoactive amines

E Bilateral hemorrhage of adrenal glands resulting in adrenal insufficiency

Answer, A Activation of innate immune cells and endothelium by microbial products.

The patient has symptoms consistent with urosepsis and systemic inflammatory

response syndrome (SIRS), which is cause by systemic activation of the innate

immune system, in this instance by microbial products.

B Anaerobic glycolysis resulting in lactic acidosis is a result, not a consequence, of

SIRS.

C Disseminated intravascular coagulation resulting in widespread hypoxia is a result,

not a consequence, of SIRS.

D Triggering of mast cells resulting in release of vasoactive amines is the underlying

cause of anaphylactic shock, not SIRS.

E Bilateral hemorrhage of adrenal glands resulting in adrenal insufficiency may be a

complication of sepsis and SIRS, not a cause.


23 A 63-year-old woman has had a fever and felt faint for the past 2 days. On physical

examination, her temperature is 38.4° C, pulse is 101/min, respirations are 17/min, and

blood pressure is 85/40 mm Hg. She has marked peripheral vasodilation. The serum

lactic acid level is 6.8 mg/dL. Which of the following laboratory findings is most likely

related to the cause of her clinical condition?

A Blood culture positive for Pseudomonas aeruginosa

B Reduced serum C3

C Elevated serum creatine kinase

D Increased blood urea nitrogen

E Reduced PO2 on blood gas measurement

Answer, A Blood culture positive for Pseudomonas aeruginosa. Her symptoms are

consistent with septic shock .

B Reduced serum C3 is seen in disorders in which there is increased activation of

complement, particularly disorders in which immune complexes form, such as

systemic lupus erythematosus.

C Elevated serum creatine kinase may be seen in disorders associated with injury to the

heart or skeletal muscle.

D Increased blood urea nitrogen is associated with renal failure, a potential complication

of SIRS.
E Reduced PO2 on blood gas measurement may be the result of many disorders,

including acute respiratory distress syndrome, one of the complications of SIRS.

Question #2
A 22-year-old woman has had multiple episodes of aspiration of food associated with
difficulty swallowing during the past year. On auscultation of her chest, crackles are
heard at the base of the right lung. A barium swallow shows marked esophageal dilation
above the level of the lower esophageal sphincter. A biopsy specimen from the lower
esophagus is most likely to show which of the following lesions?
A Absence of myenteric ganglia
B Barrett esophagus
C Esophageal atresia
D Esophageal carcinoma
E Systemic sclerosis
Correct answer: A, achalasia is characterized by the triad of incomplete lower esophageal
sphincter (LES) relaxation, increased LES tone, and esophageal aperistalsis. It causes
functional obstruction of the esophagus
B, Barrett esophagus is a complication of chronic GERD that is characterized by
metaplasia of the normal squamous esophageal epithelium to columnar epithelium,
typically with goblet cells. It is a risk factor for esophageal carcinoma
C, Esophageal atresia is a rare congenital anomaly that presents with signs of obstruction
shortly after birth
D, esophageal carcinomas are uncommon in young individuals and present with
dysphagia
E, in systemic sclerosis, there is fibrosis of the esophageal wall and associated dysphagia
Question #3
A 51-year-old man has sudden onset of massive emesis of bright red blood. On physical
examination, his temperature is 36.9° C, pulse is 103/min, respirations are 23/min, and
blood pressure is 85/50 mm Hg. His spleen tip is palpable. Laboratory studies show a
hematocrit of 21%. The serologic test result for HBsAg is positive. He has had no prior
episodes of hematemesis. The hematemesis is most likely to be a consequence of which
of the following?
A Barrett esophagus
B Candida albicans infection
C Esophageal varices
D Reflux esophagitis
E Squamous cell carcinoma

Correct answer: C. The low hematocrit suggests chronic blood loss and the positive test
for HBsAg indicates liver disease. The palpable spleen supports the diagnosis of
portal hypertension secondary to cirrhosis. One of the complications of portal
hypertension is esophageal varices, which can lead to gradual blood loss as well as
severe acute bleeding (hematemesis).
A, Barrett esophagus is an epithelial lesion of the esophagus that predisposes to
carcinoma. It does not cause bleeding.
B, C. albicans causes superficial inflammation with ulcers in the oral cavity and
esophagus. It may cause pain but does not present with sudden bleeding.
D, reflux esophagitis presents with heartburn. Sudden massive blood loss is rare.
E, squamous cell carcinoma of the esophagus causes difficulty in swallowing and may
ulcerate, with some blood loss, but rarely massive sudden hematemesis.
Question #4
A 55-year-old man has had increasing difficulty swallowing during the past 6 months. There
are no significant findings on physical examination. Upper gastrointestinal endoscopy
shows a 3 cm area of erythematous mucosa for at the gastroesophageal junction. A biopsy
specimen from the lower esophagus shows metaplasia of the squamous epithelium to
columnar epithelium with some mucin filled goblet cells. Which of the following
complications is most likely to occur as a consequence of this patient’s condition?
A Achalasia
B Adenocarcinoma
C Diverticular formation
D Lacerations (Mallory-Weiss syndrome)
E Squamous cell carcinoma

Correct answer: B. Squamous to columnar epithelial metaplasia in the lower esophagus


is diagnostic of Barrett esophagus. The greatest risk with this lesion is the
development of adenocarcinoma in the area of metaplasia.
A, achalasia is a developmental defect that leads to loss of the myenteric plexus in the
esophagus, impairing peristalsis and swallowing
C, diverticula are most commonly seen in the colon and rectum. They are not associated
with columnar metaplasia of the epithelium
D, esophageal lacerations are caused by trauma associated with severe vomiting or
retching
E, squamous cell carcinoma occurs in the esophagus, but it is not the tumor that occurs
in areas of Barrett esophagus. It typically affects the mid portion of the esophagus
Question #5
A 68-year-old man has had “heartburn” and substernal pain after meals for 25 years. For the
past year, he has had increased pain with difficulty swallowing both liquids and solids.
On physical examination, there are no remarkable findings. Upper gastrointestinal
endoscopy shows an ulcerated lower esophageal mass that nearly occludes the lumen of
the esophagus. A biopsy specimen of this mass is most likely to show which of the
following neoplasms?
A Adenocarcinoma
B Neuroendocrine tumor
C Leiomyosarcoma
D Non-Hodgkin lymphoma
E Squamous cell carcinoma

Correct answer: A. Long-standing esophageal reflux, as in this patient, is associated with


Barrett esophagus and subsequent development of adenocarcinoma in the metaplastic
epithelium.
B, Neuroendocrine tumors are usually seen in the intestines, less often in the stomach,
and rarely in the esophagus. These tumors present clinically with signs related to
production of various hormones, including catecholamines.
C, leiomyosarcoma is a rare tumor; in the GI tract, it is most common in the wall of the
stomach
D, non-Hodgkin B cell lymphoma may develop in lymph nodes in any anatomic region.
Rare forms of B cell lymphomas arise in from lymphocytes the gastric mucosa, but not
in the esophagus.
E, squamous cell carcinoma is associated with alcohol use, smoking and HPV infection,
but usually not with esophageal reflux.
Question #9

In a study that examines granuloma formation in the lung in response to infection with

Mycobacterium tuberculosis, it is observed that some cells within the granuloma express

MHC class II molecules. These class II-expressing cells are most likely derived from

which of the following peripheral blood leukocytes?

A Basophil

B CD4+ T lymphocyte

C Monocyte

D Natural killer cell

E Neutrophil

Correct answer: C. Class II MHC molecules are expressed on a few cell types, mainly

dendritic cells, macrophages and B lymphocytes. In this granulomatous inflammation,

the dominant cells are macrophages derived from blood monocytes. These macrophages

assume the appearance of epithelioid cells. A, D, E, basophils, NK cells and neutrophils

do not express class II MHC molecules and cannot be induced to do so even in

inflammatory reactions.

B, CD4+ T cells are involved in granuloma formation by secreting IFN –gamma, but they

do not give rise to class II expressing cells in the granulomas .


Question #10

Within 5 minutes after a bee sting, a 15-year-old girl suddenly has difficulty breathing, with

marked inspiratory stridor from laryngeal edema. She experiences marked urticaria and

notes swelling of the hand that was stung. Which of the following is the best

pharmacologic agent to treat her signs and symptoms?

A Cyclosporine

B Epinephrine

C Glucocorticoids

D Anti IgG antibody

E Penicillin

Correct answer: B. This patient is developing the allergic reaction known as anaphylaxis,

characterized by laryngeal edema (and fall in blood pressure). The urticaria is another

manifestation of allergy. Anaphylaxis is a medical emergency that has to be treated with

epinephrine, to contract blood vessels and dilate airways, thus counteracting the life-

threatening complications of this reaction.

A, cyclosporine is an immunosuppressive drug used to treat T cell mediated graft rejection

C, glucocorticoids are anti-inflammatory drugs used to treat many inflammatory diseases,

including autoimmune diseases and chronic allergies (such as asthma)

D, anti-IgG is not a therapeutic

E, penicillin is an antibiotic used to treat bacterial infections


Question #11

A laboratory worker who is “allergic” to fungal spores is accidentally exposed to a culture

of the incriminating fungus on a Friday afternoon. Within 1 hour, he develops bouts of

sneezing, watery eyes, and nasal discharge. The symptoms seem to subside within a few

hours of returning home, but reappear the next morning, although the laboratory fungus is

not present in his home environment. The symptoms persist through the weekend. Which

of the following cells is most likely to predominate on microscopic examination of the

patient’s nasal discharge?

A Dendritic cells

B Eosinophils

C Macrophages

D Mast cells

E NK cells

Correct answer: B. The delayed reaction is typical of the late-phase reaction in allergies.

This reaction is dominated by infiltration of leukocytes, most prominently eosinophils,

which are recruited by chemokines produced by mast cells and other cells.

A, dendritic cells are mostly tissue resident and not released into secretions

C, E, macrophages are prominent in Th1-mediated delayed hypersensitivity reactions and

NK cells in some viral infections, but not in Th2-mediated allergies

D, mast cells are activated in allergies but they are tissue-resident and not released into

secretions
Question #12

A 28-year-old man has had hemoptysis and hematuria for the past 2 days. On physical

examination, his temperature is 36.8° C, pulse is 87/min, respirations are 19/min, and

blood pressure is 150/90 mm Hg. Laboratory studies show creatinine of 3.8 mg/dL and

urea nitrogen of 35 mg/dL. Urinalysis shows 4+ hematuria, 2+ proteinuria, and no

glucose. A renal biopsy specimen examined microscopically shows glomerular damage

and linear immunofluorescence with labeled complement C3 and anti-IgG antibody.

Which of the following autoantibodies has the greatest specificity for this patient’s

condition?

A Anti–basement membrane

B DNA topoisomerase 1

C Anti–double-stranded DNA

D Anti-IgM

E Anti–SS-A

Correct answer: A. The renal manifestations indicate a mix of nephritis (hematuria) and

nephrotic syndrome (proteinuria), and the linear pattern of immunofluorescence is

characteristic of anti-glomerular basement membrane (GBM) antibody-mediated

disease. Hemoptysis suggests that the antibody has also bound to lung basement

membrane, diagnostic of the Goodpasture syndrome.

B, C, E, these autoantibodies are found in various autoimmune diseases (systemic

scelerosis, SLE and Sjogren syndrome) but not in Goodpasture syndrome


D, anti-IgM antibody is not seen in any autoimmune or allergic disease

16. A 32-year-old woman has experienced dull pelvic pain for the past 2 months. Physical

examination shows a right adnexal mass. An abdominal ultrasound scan shows a 7.5-cm

cystic ovarian mass. The mass is surgically excised. The surface of the mass is smooth,

and not adherent to surrounding pelvic structures. On gross examination, the cystic mass

is filled with hair. Microscopically, squamous epithelium, tall columnar glandular

epithelium, cartilage, and fibrous connective tissue are present and resemble normal

tissue counterparts. Which of the following is the most likely diagnosis?

A Adenocarcinoma

B Fibroadenoma

C Hamartoma

D Mesothelioma

E Teratoma

Answer, E Teratoma. The presence of tissues from multiple germ layers and of multiple

types is diagnostic of teratoma, a type of germ cell tumor. The features in this case are

classic for a mature cystic teratoma, also known as a dermoid cyst.

A Adenocarcinoma contains only malignant glands.

B Fibroadenoma contains both benign glands and benign spindled cells resembling

fibroblasts, but would not contain other elements seen in this tumor.

C Hamartoma (frequently a misnomer, because these are usually clonal neoplasms) is

composed of a benign neoplastic stromal component and reactive epithelial cells,


often gland forming. It also does not contain some of the elements seen in this

neoplasm (e.g., hair, several types of epithelium).

D Mesothelioma is composed only of the lining cells of the peritoneum or other serosal

surfaces.

17 A Pap smear obtained from a 29-year-old woman during a routine health maintenance

examination is abnormal. She is currently asymptomatic. She has a history of multiple

sexual partners. Cervical biopsy specimens are obtained and the microscopic appearance

is shown in the figure (Fig 7-7 PBD 9 remove arrow). Which of the following is the most

likely diagnosis?

A Adenocarcinoma

B Carcinoma in situ

C Dysplasia

D Squamous metaplasia

E Cervical carcinoma

Answer, E Cervical squamous cell carcinoma. This is an invasive process composed of

poorly differentiated squamous cells, some forming keratin pearls- features

compatible with invasive cervical carcinoma.

A Adenocarcinoma is associated with malignant glands, sometimes with mucus

production.

B Carcinoma in situ would be confined to the surface of the cervix without any breach

of the basement membrane; nests of invasive cells are evident here.


C Dysplasia also is confined to the surface epithelium; when it involves the entire

thickness of the cervical epithelium it is called carcinoma in situ

D Squamous metaplasia is confined to the surface epithelium and by definition does not

exhibit cellular atypia.

18. A 44-year-old woman feels painless lumps in her armpit, which were not present a

month ago. On examination, right axillary lymphadenopathy is present. The nodes are

painless but firm and immobile, seeming to be “stuck down” on surrounding soft tissue.

Which of the following is the most likely lesion in her right breast?

A Acute mastitis with abscess

B Fibroadenoma

C Infiltrating ductal carcinoma

D Carcinoma in situ

E Teratoma

Answer, C Infiltrating ductal carcinoma. Painless, firm lymph nodes that are fixed are

likely to be a malignant process. In a woman of this age, breast cancer is the most

likely diagnosis, and infiltrating ductal carcinoma is the most likely subtype.

A Acute mastitis with abscess would be expected to be associated with a painful breast

mass; moreover, the lymph nodes might be fluctuant (soft) if associated with necrosis.

B Fibroadenoma is a benign disorder and is not associated with lymph node

involvement.

D Carcinoma in situ by definition is not associated with lymphatic spread.


E Teratoma does not occur in the breast and, even if malignant in origin (e.g., from the

ovary) would not present in axillary lymph nodes.

19 A 54-year-old woman notes a lump in her right breast. Physical examination shows a 2-

cm mass fixed to the underlying tissues beneath the areola and three firm, nontender,

lymph nodes palpable in the right axilla. There is no family history of cancer. An

excisional breast biopsy is performed, and microscopic examination shows the findings in

the figure (Fig 23-22C, PBD9). Which of the following molecular abnormalities in such a

tumor can be directly exploited therapeutically?

A Amplification of the HER2 gene

B Deletion of one copy of BRCA1 gene

C Overexpression of cyclin D

D Inactivation of one BRCA2 gene copy

E Loss of E- cadherin expression

Answer, A Amplification of the HER2 gene. The image shows a poorly differentiated

carcinoma. Amplification of HER2 is a relatively common feature in poorly

differentiated breast carcinoma and the product of the gene, a receptor tyrosine kinase

that is a member of the EGF receptor family, can be targeted with inhibitory

antibodies or small molecule drugs.

B Deletion of one copy of BRCA1 gene is associated with poorly differentiated breast

carcinomas that are often “triple negative”, but this lesion is not targetable.
C Overexpression of cyclin D. Cyclin D1 is not the target of therapy; rather, enzymes

that are activated by cyclin D1 (CDK4 and CDK6) are the targets of drug therapy.

D Inactivation of one BRCA2 gene copy is associated with breast carcinoma, but this

lesion is not targetable.

E Loss of E- cadherin expression is associated with lobular carcinoma and is not

therapeutically targetable.

20 The mother of a 5-year-old boy notices that his abdomen has enlarged in the past 6

months. On physical examination there is an ill-defined abdominal mass. An abdominal

CT scan shows a 9-cm mass in the region of the right adrenal gland. The mass is removed

and microscopically shows primitive hyperchromatic cells. Cytogenetic analysis of tumor

cells shows many double minutes and homogeneously staining regions. Which of the

following genes is most likely to have undergone alterations to produce these findings?

A ABL

B BCL2

C KRAS

D NMYC

E. RB

Answer, D NMYC. One of the most common solid tumors in pediatric age groups is

neuroblastoma, an aggressive neoplasm composed of proliferating neuroblasts. A

subset of these tumors has amplification of the NMYC oncogene. The amplified

chromosomal region may be found within a chromosome and appear as


homogeneously staining regions or may be extrachromasomal and appear as

numerous small chromosomal fragments known as double minutes.

A ABL is usually converted to an oncogene by chromosomal rearrangements that create

a fusion gene encoding a chimeric ABL protein that is constitutive active (e.g., BCR-

ABL).

B BCL2 is usually converted to an oncogene by chromosomal rearrangements that place

BCL2 next to a strong enhancer and drive high-level BCL2 protein expression. Such

an event occurs in follicular lymphomas

C KRAS is typically activated by point mutations in several cancers , not in

neuroblastoma.

E. RB is a tumor suppressor gene that is lost through disruptive point substitutions or

deletions. It is not amplified in any tumor

1 A female infant born at term shows failure to thrive and failure to achieve developmental

milestones. A pedigree reveals only this child is affected out of four generations on both

sides of the family. Tissue fibroblasts obtained from this child shows a 46,XX karyotype.

Cultured fibroblasts show absence of the enzyme phenylalanine hydroxylase. What is the

most likely recurrence risk for this condition in siblings of this infant?

A 3%

B 8%

C 15%

D 25%

E 50%
Answer, D: Phenlalanine hydroxylase deficiency is the cause of phenylketonuria and is

an autosomal recessive disorder in which heterozygotes are asymptomatic carriers.

The most likely scenario is that both parents are heterozygotes; based on random

assortment of gametes, each child has a 1 in 4 chance of being homozygous for the

trait. 50% probability of being affected (E) would correspond to an autosomal

dominant trait, which would mean that one of the parents would be affected. Other

probabilities of being affected of less than 25% (A, B, C) would hold for traits with

incomplete penetrance, but phenylalanine hydroxylase deficiency is a highly

penetrant trait.

2 A 22-year-old man has a sudden loss of vision in the right eye. On physical examination,

there is a subluxation of the lens in the right eye. An echocardiogram shows a floppy

mitral valve and dilated aortic arch. The patient’s brother and his cousin are similarly

affected. He is prescribed β-adrenergic and angiotensin receptor blocking drugs. A

mutation affecting the gene encoding which of the following proteins is most likely to be

present in this patient?

A Collagen

B Dystrophin

C Fibrillin

D NF1 protein

E Spectrin
Answer, C: The phenotype ( subluxation of lens, floppy mitral valve and aortic

aneurysm) and pattern of inheritance (autosomal dominant) is classic for Marfan

syndrome, which is cause by mutations in fibrillin-1, a structural protein that contributes

to the elasticity of tissues such as the aorta, heart valves, and support structures of the

lens. The other proteins listed are mutated in other disorders:

A, collagen, Ehlers-Danlos syndrome and related disorders

B, dystrophin, muscular dystrophy

D, NF1, neurofibromatosis-1

E, spectrin, hereditary spherocytosis and other red cell disorders

3 An 11 year old child has exhibited poor wound healing, even with minor trauma since

infancy. On examination she has hyperextensible joints and fragile, extremely stretchable

skin. A diaphragmatic hernia was repaired soon after birth. One parent and one of three

siblings are also affected. A mutation in a gene encoding for which of the following type

of proteins is most likely causing this child’s disease?

A Enzyme

B Growth regulation

C Ion channel

D Receptor

E Structural protein

Answer, E. The phenotype and pattern of inheritance is most consistent with an

autosomal dominant disorder that affects the stability of joints and soft tissues, such
as an inherited disorder of collagen. In general, germline defects affecting structural

proteins are likely to exhibit autosomal dominant patterns of inheritance.

A, defects in enzymes are typically associated with autosomal recessive patterns of

inheritance

B, defects in growth regulation may be inherited in an autosomal dominant fashion, but

typically are associated with alterations in the size of tissues or cells and sometimes

with neoplasia

C, D, defects in signaling pathways (receptors, growth factor signaling pathways) and in

ion channels may have autosomal dominant patterns of inheritance, but do not result

in the phenotype observed in this pedigree. Familial hypercholesterolemia is an

example of mutation in LDL receptor and cystic fibrosis is an example of mutation in

an ion channel

4 A 4-year-old girl has sudden onset of right hip pain. On examination, the child’s right hip

is dislocated. The child can bend her thumb backward to touch the forearm. Her skin is

noted to be extraordinarily stretchable. Radiographs of her spine show marked lateral and

anterior curvature. A sibling is similarly affected, and her parents are normal. Laboratory

investigations reveal reduced levels of lysyl hydroxylase. Which of the following is the

most likely cause for this child’s findings?

A Congenital syphilis with involvement of the bones

B Defect in maturation of collagen

C Defect in vitamin D synthesis


D Deficient synthesis of collagen alpha chains

E Defect in Vitamin C synthesis

Answer, B. This appears to be an autosomal recessive disorder that is affecting collagen

or other structural support molecules. Lysyl hydroxylase is an enzyme that introduces

crosslinks into collagen that stabilize collagen fibrils and as with most enzyme defects,

only causes disease if individuals inherit two defective alleles.

A, Syphilis is an acquired disorder and the phenotype is not consistent with congenital

syphilis, nor is involvement in a sibling.

C, vitamin D abnormalities are mainly associated with bone defects (rickets and

osteomalacia) and metabolic disturbances of calcium and phosphate.

D, collagen abnormalities could cause a similar phenotype, but would follow an

autosomal dominant mode of inheritance as is the case with inherited disorders affecting

structural proteins.

E, vitamin C deficiency, which leads to defective collagen crosslinking, is typically

caused by poor diet (scurvy); interestingly, lysyl oxidase require vitamin C for its activity,

but the phenotype described here does not match that associated with scurvy

5. A clinical study is undertaken with subjects from families in which complications of

atherosclerotic cardiovascular disease and tendinous xanthomas occurred before age 30

years. The affected individuals benefit from treatment with pharmacologic agents that

inhibit HMG-CoA reductase. Affected individuals in these families are most likely to
have a mutation in a gene encoding cell surface receptor for which of the following

molecules?

A Triglyceride

B Free fatty acids

C LDL cholesterol

D Insulin

E HDL cholesterol

Answer, C. The clinical phenotype is consistent with familial hypercholesterolemia

( early onset atherosclerosis), an autosomal dominant disorder caused by defects in

the LDL receptor. Because of the resulting defect in LDL uptake, hepatocyte

synthesis of cholesterol is increased, leading to high levels of circulating LDL. The

rate-limiting step in cholesterol synthesis is catalyzed by HMG-CoA reductase, thus

the efficacy of HMG-CoA reductase inhibitors in this disorder.

A, B, triglycerides are hydrolyzed by lipoprotein lipase and taken up into cells as free

fatty acids in a receptor-independent fashion.

D, insulin receptor defects are a rare cause of insulin resistance and diabetes

E, HDL cholesterol is cleared by receptors, but high levels of HDL are not associated

with cardiovascular disease and may be protective.

18. A 33-year-old man has smoked two packs of cigarettes per day since he was a teenager.

He has had painful thromboses of the superficial veins of the lower legs for 1 month.

Over the next year, he develops chronic, poorly healing ulcerations of his feet. One toe
becomes gangrenous and is amputated. Histologically, at the resection margin, there is an

acute and chronic vasculitis involving medium-sized arteries, with segmental

involvement. Antibodies against which of the following antigens are most likely involved

in the pathogenesis of this disease

A Myeloperoxidase

B Tobacco smoke

C Vascular basement membrane

D Hepatitis B surface antigen

E Nucleolar antigens

Answer, B, Tobacco smoke. The clinicopathologic features are consistent with

thromboangiitis obliterans, which is a severe peripheral vasculitis that is confined to

smokers. The pathogenesis is uncertain but the tight association with smoking has led

to the hypothesis that the disease is caused by an immune response to antigens present

in tobacco smoke.

A, antibodies to myeloperoxidase in neutrophils is one form of anti-neutrophil antibody

that is associated with certain vasculitides, particularly granulomatosis and

polyangiitis and microscopic angiitis.

C, antibodies to vascular basement membrane caused Goodpasture syndrome,

characterized by severe renal and pulmonary disease.

D, antibodies to hepatitis B surface antigen underlie some cases of polyarteritis nodosa

and also may give rise to membranous glomerulonephritis


E, antibodies to nucleolar antigens are associated with a large number of different auto-

immune disorders

19. A 35-year-old woman with a history of injection drug use has developed a high fever

over the past day. On examination her temperature is 37.6° C. A heart murmur is

auscultated. Blood culture is obtained and grows Staphylococcus aureus. She develops a

severe headache. A head CT scan shows an intracranial hemorrhage. Which of the

following vascular complications has most likely occurred in her brain?

A Granulomatous vasculitis

B Hyperplastic arteriolosclerosis

C Atherosclerosis

D Mycotic aneurysm

E Immune complex small vessel vasculitis

Answer, D, Mycotic aneurysm. The clinicopathologic features are consistent with S.

aureus infective endocarditis and seeding of cerebral vessels, with rupture secondary

to mycotic aneurysm formation.

A, granulomatous vasculitis cannot account for the S. aureus sepsis.

B, hyperplastic arteriolosclerosis is associated with hypertension, for which there is no

evidence.

C, atherosclerosis is more likely to be associated with thrombosis and non-hemorrhagic

stroke, and also is unlikely as a cause of a cerebrovascular accident in a 35 year old

woman.
E, immune complex small vessel vasculitis is more likely to present with purpura and

renal disease than as hemorrhagic stroke

20 A 69-year-old woman has been bedridden while recuperating from a bout of viral

pneumonia complicated by bacterial pneumonia for the past 2 weeks. Physical

examination now shows some swelling and tenderness of the right leg, which worsens

when she raises or moves the leg. Which of the following terms best describes the

condition involving her right leg?

A Disseminated intravascular coagulation

B Lymphedema

C Thromboangiitis obliterans

D Thrombosis of deep veins

E Varicose veins

Answer, D, Thrombosis of deep veins is the most likely diagnosis in a bedridden patient,

particularly in those past the age of 50 years.

A, disseminated intravascular coagulation usually occurs secondary to a severe systemic

disorder and if acute is more likely to present with bleeding than thrombosis

B, lymphedema usually occurs due to some local event (e.g., surgery, trauma) that

compromises lymphatic drainage

C, thromboangiitis obliterans usually presents with pain and sometimes gangrene in the

distal extremity of a smoker


E, varicose veins are usually superficial and evident to the eye; they tend to cause pain

instead of swelling.

21 A 10-year-old boy has a 2-cm spongy, dull red, circumscribed lesion on the upper outer

left arm. The parents state that this lesion has been present since infancy, and the

appearance has not appreciably changed. The lesion is excised, and its microscopic

appearance is shown in the figure (Fig 11-30 C, PBD9). Which of the following is the

most likely diagnosis?

A Angiosarcoma

B Hemangioma

C Kaposi sarcoma

D Lymphangioma

E Vasculitis

Answer, B, Hemangioma. The lesion consists of dilated vessels lined by normal-

appearing endothelium, consistent with a benign hemangioma.

A, Angiosarcoma is a vascular lesion of older adults in which vascular spaces are lined

with malignant-appearing endothelial cells.

C, Kaposi sarcoma is a vascular lesion that is confined to severely immunodeficient

patients (e.g., individuals with late-stage untreated HIV infection [AIDS]) or elderly

individuals, typically beyond the age of 70 years.

D, lymphangioma in children usually occurs in the head and neck region (cystic

hygroma) and is more common in girls with Turner syndrome (45, X karyotype)
E, Vasculitis usually causes an acute or subacute systemic illness. In vasculitis vessel

walls show inflammation.

22 A 35-year-old man is known to have been HIV-positive for the past 10 years, but has

refused treatment with antiviral drugs. Physical examination now shows multiple reddish

purple, nodular skin lesions comprised of spindle shaped cells with slit like spaces

containing red cells. These lesions have been slowly increasing for the past year. Which

of the following infectious agents is most likely to play a role in the pathogenesis of these

skin lesions?

A Human herpes virus 8

B Epstein-Barr virus

C Hepatitis C virus

D Mycobacterium avium complex

E Herpes simplex virus 2

Answer, A, Human herpes virus 8. The findings are consistent with Kaposi sarcoma, an

immunodeficiency-related endothelial proliferation in which the tumor cells are

infected with human herpes virus 8. In advanced lesions (nodular stage) proliferating

spindle cells are dominant.

B, Epstein-Barr virus in the context of HIV infection is most likely to be associated with

an aggressive B cell lymphoma

C, Hepatitis C virus is non-transforming and among neoplasms is associated only with

hepatocellular carcinoma and some low-grade B cell lymphomas.


D, Mycobacterium avium complex infection tends to mainly produce disease in the lung

and does not induce vascular proliferations

E, Herpes simplex virus 2 infection is mainly associated with genital herpes, but on

occasion can produce disseminated disease in patients with AIDS. It does not cause

vascular lesions, however.

1 A 66-year-old man has had cough and worsening shortness of breath for 3 years. On

physical examination, pulse is 77/min and BP is 110/80 mm Hg. Ejection fraction is

normal and he does not have anginal pain. His liver span is increased to 14 cm. He has

pitting edema to his knees. Jugular venous distention is noted to the angle of the jaw at

45-degree elevation of his head while lying down. Which of the following is most likely

causing his heart disease?

A Anemia

B Essential hypertension

C Aortic stenosis

D Systemic Lupus erythematosus

E Chronic obstructive lung disease

Answer, E. The findings in this patient are consistent with isolated right-sided heart

failure (cor pulmonale) secondary to chronic obstructive pulmonary disease (COPD).

Chronic lung disease associated with poor ventilation of alveoli results in

vasoconstriction and pulmonary hypertension, which may over time lead to right-sided

heart failure.
A, Anemia can lead to high output cardiac failure if severe, but cannot explain the history

of cough.

B, C, Essential hypertension and aortic stenosis often cause left-sided heart failure.

D, Systemic lupus erythrematosus sometimes produces interstitial lung disease, but is a

much less frequent cause of lung disease and secondary heart failure than COPD.

2 A 62-year-old woman has had increasing dyspnea for the past 2 years. She now awakens

at night with air hunger and cough productive of frothy sputum. On examination, she has

rales in all lung fields. Echocardiography shows a decreased ejection fraction of 30%

with concentric increase in left ventricular wall size. The valves appear normal. Which of

the underlying diseases does she most likely have?

A Amyloidosis

B Pulmonary fibrosis

C Hypertension

D Obstructive sleep apnea

E Pericarditis

Answer, C. Concentric left ventricular hypertrophy is a typical response to chronic

hypertension and pressure overload of the heart, leading eventually to left-sided heart

failure, high pulmonary vein pressures, pulmonary edema and cough.

A, Cardiac amyloidosis produces restrictive heart disease, sometimes accompanied by

arrhythmia due to conduction defects. It is not associated with ventricular hypertrophy

or systolic failure
B, D, Pulmonary fibrosis and obstructive sleep apnea, as with other lung disorders

associated with hypoxemia, may result in right-sided heart disease.

E, Pericarditis, if chronic can produce a restrictive cardiac picture and is not associated

with left ventricular hypertrophy.

3 A 41-year-old woman has been awakened at night with“air hunger”for the past year. She

notes sleeping better while sitting up in bed. She does not have liver or spleen

enlargement. Chest CT scans suggest bilateral pulmonary edema. Her serum natriuretic

peptide is >400 pg/mL (very high). What cardiac disease best explains her condition?

A Pulmonary thromboebolism

B Fibrinous pericarditis

C Giant cell myocarditis

D Libman-Sacks endocarditis

E Rheumatic mitral stenosis

Answer, E. The findings in this patient are all consistent with left-sided heart failure

associated with atrial dilation (elevation in level of atrial natriuretic peptide) and

pulmonary edema, all of which are classic findings in mitral stenosis.

A, Pulmonary thromboembolism, if recurrent, may lead to right-sided heart failure.

B, Fibrinous pericarditis usually is associated with pain and friction rubs but does not

produce heart failure.


C, Giant cell myocarditis may produce heart failure associated with cardiac dilation, but

usually presents acutely or subacutely and often is associated with other findings,

such as chest pain or arrhythmia.

D, Libman-Sacks endocarditis is often associated with systemic lupus erythematosus

and usually presents with valvulitis leading to thrombi formation. It may lead to

thromboembolism and/or valve regurgitation, but usually does not produce stenotic

valve lesions.

4 A 50-year-old man has had increasing abdominal discomfort and swelling of his legs for

the past 2 years. He has smoked cigarettes for 35 years. On physical examination, he has

jugular venous distention, even when sitting up. The liver is enlarged and tender and can

be palpated 10 cm below the right costal margin. Pitting edema is observed on the lower

extremities. A chest radiograph shows bilateral diaphragmatic flattening, pleural

effusions, and increased lucency of lung fields. Which of the following is the most

common cause of these clinical features?

A Idiopathic pulmonary fibrosis

B Chronic obstructive pulmonary disease (COPD)

C Primary pulmonary hypertension

D Pulmonary valve stenosis

E Tricuspid valve stenosis

Answer, B. Smoking commonly leads to COPD, frequently with both chronic bronchitis

and emphysema. The chronic bronchitis often produces hypoxia, leading to


pulmonary hypertension and eventual right-sided heart failure, which is evident in

this patient.

A, Idiopathic pulmonary fibrosis may be associated with smoking, but would not

produce the overinflated lungs that are evident in this patient, presumably due to

emphysematous changes.

C-E, these entities are rare causes of right heart failure and are not smoking-related.

5 An infant born at term is noted to have cyanosis during the first week of life. On

examination a heart murmur is auscultated. Abnormal findings with echocardiography

include an overriding aorta and pulmonic stenosis. This infant is likely to have which of

the following additional heart defects?

A Atrial septal defect

B Ventricular septal defect

C Coarctation of the aorta

D Ventricular aneurysm

E Patent ductus arteriosus

Answer, B. Cyanosis indicates the present of a significant right to left shunt, which in the

context of overriding aorta and pulmonic stenosis is likely to be caused by tetralogy of

Fallot: overriding aorta, pulmonic stenosis, ventricular septal defect, and right ventricular

thickening.

A, atrial septal defects are usually isolated and do not produce cyanosis
C, coarctation of the aorta, depending on level, is associated with difference in blood

pressure above and below the level of the coarctation.

D, ventricular aneurysms are acquired lesion, usually due to myocardial infarction.

E, patent ductus arteriosus causes left to right shunting and is not a part of the tetralogy of

Fallot

1. A healthy 19-year-old woman suffered blunt abdominal trauma in a motor vehicle

accident. On admission to the hospital, her initial hematocrit was 33%, but over the next

hour, it decreased to 28%. She was taken to surgery, where a liver laceration was

repaired, and 1 L of bloody fluid was removed from the peritoneal cavity. She remained

stable. A CBC performed 7 days later is most likely to show which of the following

morphologic findings in the peripheral blood?

A Basophilic stippling of red cells

B Hypochromic red cells

C Leukoerythroblastosis

D Reticulocytosis

E Schistocytosis

Answer, D. The sudden appearance of anemia due to bleeding decreases oxygen

delivery to the kidney, resulting in hypoxia that is “sensed” by interstitial cells near

the proximal renal tubules. Hypoxia stabilizes the transcription factor HIF, which
upregulates production and secretion of the hormone erythropoietin. This in turn

increases the proliferation and survival of red cell progenitors in the bone marrow,

resulting in increased production of red cells (reticulocytes) in 5-7 days.

A, Basophilic stippling of red cells is a finding that can be seen in a variety of disorders

marked by disturbed erythropoiesis, including thalassemia and lead poisoning

B, Hypochromic red cells are indicative of inadequate hemoglobin production and are

most commonly seen with iron deficiency and thalassemia; red cells produced in

response to acute blood loss have normal amounts of hemoglobin and are hence

called normochromic

C, Leukoerythroblastosis refers to the presence of immature erythroid and myeloid

elements in the peripheral blood and is usually caused by marrow fibrosis, which can

be induced by metastatic cancer, granulomatous disease, or certain blood cancers.

E, Schistocytosis refers to red cell fragments that are produced by high shear stresses,

usually due to intravascular thromobosis, as may occur in disseminated intravascular

coagulation (DIC), hemolytic uremic syndrome (HUS), or thrombotic

thrombocytopenia purpura (TTP)

2 A 65-year-old man has experienced worsening fatigue for the past 5 months. On physical

examination, he is afebrile and has a pulse of 91/min, respirations of 18/min, and blood

pressure of 105/60 mm Hg. There is no organomegaly. A stool sample is positive for

occult blood. Laboratory findings include hemoglobin of 5.9 g/dL, hematocrit of 18.3%,
MCV of 72 µm3, platelet count of 250,000/mm3, and WBC count of 7800/mm3. The

reticulocyte concentration is 2.0%. A bone marrow biopsy specimen shows mild

erythroid hyperplasia. Laboratory testing will most likely reveal which of the following?

A Elevated serum ferritin

B Low serum iron

C Elevated serum hepcidin

D Increased HbA2

E Low serum transferrin

Answer, B. The findings are consistent with iron deficiency anemia, brought on and

exacerbated by bleeding.

A, ferritin is the storage form of iron ( found in liver, spleen, and bone

marrow ).Plasma ferritin is derived primarily from the storage pool of iron and hence

in iron deficiency anemia, serum ferritin is low

C, hepcidin regulates intestinal absorption of iron; high hepcidin levels in plasma

inhibit iron absorption and conversely low hepcidin levels increase iron absorption; in

iron deficiency anemia, serum hepcidin is low

D, in iron deficiency anemia, HbA2 is normal, unlike in beta-thalassemia, in which it is

elevated

E, in iron deficiency anemia, serum transferrin is elevated rather than being low.

3 During the past 6 months, a 25-year-old woman has noticed a malar skin rash that is made

worse by sun exposure. She also has had arthralgias and myalgias. On physical
examination there is erythema of skin over the bridge of the nose. Mild splenomegaly is

noted. Laboratory findings include positive serologic test results for ANA and double-

stranded DNA, hemoglobin of 8.1 g/dL, hematocrit of 24.4%, reticulocyte count of 5%,

MCV of 99 µm3, platelet count of 187,000/mm3, and WBC count of 3950/mm3. Tests of

hemostasis are normal. Which of the following laboratory findings is most likely to be

present?

A Decreased serum haptoglobin

B Decreased serum ferritin

C Elevated D dimer

D Elevated hemoglobin F

E Hemoglobinuria

Answer, A. The physical findings are consistent with systemic lupus erythematosus, an

autoimmune disorder characterized by production of antibodies against self-antigens.

In some instances, these antibodies are directed against red blood cells and act as

opsonins, producing autoimmune hemolytic anemia associated with reticulocytosis

and varying degrees of splenomegaly. Although the hemolysis is extravascular,

hemoglobin tends to “leak” into the blood from phagocytes that have consumed

antibody-coated red cells, and this hemoglobin is sopped up by haptoglobin, which is

rapidly removed by the liver.

B, ferritin is an acute phase reactant, and in the presence of inflammation it is likely to

be normal or high
C, elevated D dimer is seen with intravascular thrombosis, for which there is no

evidence

D, elevated hemoglobin F is seen in certain forms of thalassemia and in some patients

with hemoglobinopathies, neither of which are in the differential diagnosis

E, hemoglobinuria is a marker of intravascular hemolysis, which is seen with only

unusual types of autoantibodies, toxic or mechanical exposures that lyse red cells, and

paroxysmal nocturnal hemoglobinuria, an acquired disorder caused by the presence of

red cells that are hypersensitive to complement.

4 A 28-year-old woman has had easy fatigability since childhood. On physical examination,

she is afebrile and has a pulse of 80/min, respirations of 15/min, and blood pressure of

110/70 mm Hg. The spleen tip is palpable, but there is no abdominal pain or tenderness.

Laboratory studies show hemoglobin of 11.7 g/dL, platelet count of 159,000/mm3, and

WBC count of 5390/mm3. The peripheral blood smear shows a predominant population

of round red cells that lack a zone of central pallor mixed with a smaller but readily

noticeable population of larger red cells with a bluish tinge. An inherited abnormality in

which of the following RBC components best accounts for these findings?

A α-Globin chain

B β-Globin chain

C Glucose-6-phosphate dehydrogenase

D Heme porphyrin ring

E Spectrin, a membrane skeletal protein


Answer, E. The lifelong symptomatology, mild anemia, splenomegaly, and presence of

spherocytes in the blood are all consistent with hereditary spherocytosis (HS), an

autosomal dominant disorder caused by defects in the red blood cell membrane skeleton.

Several different components of the membrane skeleton may be mutated in HS, but the

common feature is that they all lead to a deficiency of spectrin, the major red cell

membrane protein, that destabilizes the lipid bilayer, causing red cells to lose membrane

as the age and become spherocytic.

A, B, defects of α-Globin or β-Globin chains are most commonly associated with

thalassemia, a disorder of ineffective red cell production. The larger red cells with a

bluish tinge are consistent with reticulocytes, suggesting that this patient has a

disorder of red cell destruction rather than production.

C, Glucose-6-phosphate dehydrogenase deficiency is associated with periodic red cell

hemolysis following infections or exposure to oxidant drugs or certain foods. It is X-

linked and rarely occurs in women.

D, Heme porphyrin ring defects are associated with porphyria or inherited sideroblastic

anemia. Porphyria usually presents with findings related to skin abnormalities or

neural dysfunction, whereas sideroblastic anemia is an underproduction anemia that

would not be associated with reticulocytosis.

5. A 13-year-old African American boy has the sudden onset of severe abdominal pain and

cramping accompanied by chest pain, nonproductive cough, and fever. On physical


examination, his temperature is 39° C, pulse is 110/min, respirations are 22/min, and

blood pressure is 80/50 mm Hg. Laboratory studies show a hematocrit of 18%. The

peripheral blood smear is shown in the figure (Fig 12.3 B BP10). A chest radiograph

shows bilateral pulmonary infiltrates. Which of the following is the most likely

mechanism for initiation of his pulmonary problems?

A Chronic hypoxia of the pulmonary parenchyma

B Parvovirus B19 infection

C Obstruction of small vessels

D Formation of autoantibodies to alveolar basement membrane

E Intravascular antibody-induced hemolysis

Answer, C. The clinical scenario is consistent with sickle cell disease complicated by

acute chest syndrome, in which sickle cells adhere to and obstruct small vessels in the

lung. This in turn sets in motion a vicious cycle, in which obstruction of pulmonary

vessels causes increasingly severe hypoxia, further exacerbating sickling and vascular

obstruction. Treatment often consists of exchange transfusion, in which sickle red

cells are replaced with normal red cells.

A, chronic hypoxia of the pulmonary parenchyma seems inconsistent with the clinical

story of acute onset symptoms

B, Parvovirus B19 infection of erythroblasts can occur in sickle cell anemia; it is

associated with aplastic crisis and rapid worsening of anemia, not pain or pulmonary

infiltrates
D, Formation of autoantibodies to alveolar basement membrane is associated with

Goodpasture syndrome which may produce radiologic changes in the lungs; patients

have hemoptysis and usually do not have severe anemia

E, Intravascular antibody-induced hemolysis would be associated with hemoglobinuria

and would not produce the observed pulmonary findings.

34 A 47-year-old woman known to be HIV-positive has had a decreasing CD4+ lymphocyte

count for the past year despite antiretroviral therapy. She has developed a fever with

nonproductive cough over the past week. On auscultation of her chest fine crackles are

present in both lungs. A chest radiograph shows infiltrates in both lungs. A transbronchial

biopsy is obtained and the microscopic appearance is shown in the figure (Fig 8-12

PBD9). Which of the following organisms is most likely responsible for this woman’s

pulmonary symptoms?

A Candida albicans

B Cryptococcus neoformans

C Cytomegalovirus

D Klebsiella pneumoniae

E Pneumocystis jiroveci

Answer, C. Individuals with HIV and a deficiency of CD4+ T cells are susceptible to a

variety of opportunistic infections, including candida, cryptococcus, pneumocystis, and

cytomegalovirus. Of these, those that are most likely to involve the lung are

cytomegalovirus and pneumocystis. The image shows the characteristic cytopathic


features of cytomegalovirus infection-infected cells are enlarged and contain prominent

intranuclear basophilic inclusions.

A, Candida is a fungus that most often presents with oropharyngeal (thrush) or

esophageal infection.

B, Cryptococcus is a fungus that often presents with CNS involvement.

D, Klebsiella is a bacterium that produces a pneumonia that often is associated with

copious production of mucoid sputum.

E, Pneumocystis is a fungus that causes pneumonia with pink staining foamy intra-

alveolar infiltrates in H&E stained sections. The organisms are better visualized with

special stains for fungus.

35 A previously healthy 43-year-old woman has had malaise and an 8-kg weight loss over

the past 3 years. She has had fever and a nonproductive cough with increasing dyspnea

for the past 3 days. On physical examination, her temperature is 37.8° C. There is

dullness to percussion over the lungs and diffuse crackles on auscultation. A chest

radiograph shows extensive bilateral infiltrates. Bronchoalveolar lavage is done, and the

fluid is stained with Gomori methenamine silver, with high-power microscopic

appearance shown in the figure (Fig 13-41B BP10). Which of the following underlying

conditions is most likely present in this woman?

A Centrilobular emphysema

B Diabetes mellitus

C HIV infection
D Sarcoidosis

E Severe combined immunodeficiency

Answer, C. The bronchoalveolar lavage contains round to cup shaped organisms that are

stained black with the silver stain. This appearance is characteristic of Pneumocystis

jiroveci. Pneumocystis infection is confined to individuals with severe T cell

immunodeficiency; in this previously healthy adult woman, the most likely cause is

unrecognized HIV infection.

A, B, D, sarcoidosis, diabetes mellitus, and centrilobular emphysema are not associated

with T cell immunodeficiency and do not lead to an increased risk of Pneumocystis

infection.

E, severe combined immunodeficiency could be associated with Pneuomocystis infection,

but typically presents early in life with recurrent infections.

36 A 64-year-old man, who is a chain-smoker, has had a cough and a 5-kg weight loss over

the past 3 months. Physical examination shows clubbing of the fingers. He is afebrile. A

chest radiograph shows no hilar adenopathy, but there is cavitation within a 3-cm lesion

near the right hilum. Laboratory studies show serum calcium of 12.3 mg/dL, phosphorus

of 2.4 mg/dL, and albumin of 3.9 g/dL. Bronchoscopy shows a lesion almost occluding

the right main bronchus. Examination of the biopsy suggests that this tumor could be

treated effectively by immunotherapy with checkpoint inhibitors. Which of the following

neoplasms is most likely to be present in this patient?

A.Bronchial carcinoid
B Metastatic renal cell carcinoma

C Non-Hodgkin lymphoma

D Small cell carcinoma

E Squamous cell carcinoma

Answer, E. A bronchial lesion associated with cavitation, clubbing of the fingers, and

hypercalcemia is likely to be squamous cell carcinoma, which often produces parathyroid

hormone-related protein (PTHrP) leading to paraneoplastic hypercalcemia. Checkpoint inhibitors

are active in smoking-related cancers such as pulmonary squamous cell carcinoma, possibly

because of the high mutational burden induced by exposure to carcinogens in tobacco smoke.

The larger the number of mutations, higher is the likelihood that neo antigens that can be

recognized by T cells are present in the tumor.

A, bronchial carcinoids are usually well circumscribed, do not cavitate, and are not associated

with hypercalcemia or clubbing. They may produce hormones that lead to carcinoid syndrome

(flushing, diarrhea, bronchoconstriction) or other paraneoplastic phenomena.

B, metastatic renal cell carcinoma would most often present as a mass in peripheral lung fields

rather than the hilum of the lung.

C, non-Hodgkin lymphoma is more likely to present with lymphadenopathy, and usually is not

associated with hypercalcemia or clubbing.

D, small cell carcinoma is most often associated with paraneoplastic syndromes caused by

production of ACTH leading to Cushing syndrome; production of antidiuretic horomone (ADH),

leading to the syndrome of inappropriate ADH production; or induction of autoantibodies that

lead to neuromuscular dysfunction. It is aggressive and usually has spread to lymph nodes by the
time of diagnosis. It initially responds well to treatment with chemotherapy; immune checkpoint

inhibitors have not been approved to treat small cell carcinoma.

37 A 55-year-old woman, a nonsmoker, has had a chronic nonproductive cough for 6

months along with 8-kg weight loss. On physical examination there are no remarkable

findings. Her chest radiograph shows a right peripheral subpleural mass. A fine-needle

aspiration biopsy is performed, and she undergoes a right lower lobectomy. The

microscopic examination of the lesion shows irregular glands invading the surrounding

lung. Which of the following molecular test findings is likely to be most useful in

deciding if her cancer may benefit from therapy targeting a tyrosine kinase?

A Amplification of FGFR1 gene

B Inactivation of CDKN2A gene

C Loss of both copies of TP53

D Amplification of MYC gene

E Rearrangement of ALK gene

Answer, E. The microscopic description is that of adenocarcinoma, which may be

associated with mutations in KRAS, TP53, and CDKN2A, as well as rearrangements

involving ALK. ALK rearrangements create a chimeric gene encoding a constitutively

active ALK tyrosine kinase, and treatment of such cancers with ALK inhibitors often

results in clinical benefit.


A, FGFR1 amplifications are most commonly associated with squamous cell carcinoma.

FGFR1 encodes a tyrosine kinase and inhibitors of FGFR1 are being studied in the clinic,

but are not yet in clinical use..

B, TP53 is a tumor suppressor gene that encodes a transcription factor that drives the

expression of genes that regulate cell cycle progression, DNA repair and apoptosis. Loss

of P53 generally indicates a poor prognosis (regardless of tumor type), but is not

currently targetable.

C, CDKN2A is a gene that encodes two tumor suppressors, ARF (a positive regulator of

TP53 function) and p16, a cyclin-dependent kinase inhibitor that acts as a brake on the

cell cycle.

D, MYC is an oncogene that encodes a transcription factor which regulates cellular

metabolism. It is frequently activated by amplification of translocation, particularly in B

cell lymphomas. It is not currently targetable.

38 A 60-year-old woman has had a chronic nonproductive cough for 4 months along with

loss of appetite and a 6-kg weight loss. She does not smoke. On physical examination

there are no remarkable findings. Her chest radiograph shows a right peripheral

subpleural mass. A fine-needle aspiration biopsy is performed, and she undergoes a right

lower lobectomy. She receives treatment with an inhibitor of epithelial growth factor

receptor (EGFR) and remains symptom-free for the next 5 years. Which of the following

neoplasms did she most likely have?

A Adenocarcinoma
B Bronchial carcinoid

C Malignant mesothelioma

D Small cell carcinoma

E Squamous cell carcinoma

Answer, A. EGFR mutations that produce constitutive activation of the receptor’s

tyrosine kinase activity are commonly associated with lung adenocarcinoma.

Adenocarcinoma also is relatively common in female nonsmokers and frequently occurs

as a peripheral lung nodule.

B, bronchial carcinoid is a neuroendocrine tumor that is not associated with EGFR

mutations.

C, malignant mesothelioma presents with pleural disease rather than a lung mass and is

not associated with EGFR mutations.

D, small cell carcinoma usually presents near the hilum of the lung and is associated with

TP53 and RB mutations and not with EGFR mutations.

E, squamous cell carcinoma is associated with FGFR1 amplification rather than EGFR

mutations.

39 A 61-year-old man with a long history of smoking has developed truncal obesity, back

pain, and skin that bruises easily over the past 5 months. On physical examination, he is

afebrile, and his blood pressure is 160/95 mm Hg. A chest radiograph shows an ill-

defined, 4-cm mass involving the left hilum of the lung. Cytologic examination of

bronchial washings from bronchoscopy shows round tumor cells that have the appearance
of lymphocytes but are larger. The patient is told that, although his disease is apparently

localized to one side of the chest cavity, surgical treatment is unlikely to be curative. He

also is advised to stop smoking. Which of the following neoplasms is most likely to be

present in this patient?

A Adenocarcinoma

B Malignant mesothelioma

C Non-Hodgkin lymphoma

D Small cell carcinoma

E Squamous cell carcinoma

Answer, D. The patient has Cushing syndrome, a paraneoplastic syndrome. In this case,

based on the reported appearance of the tumor cells, it is caused by production of ACTH

by small cell carcinoma. Among lung carcinomas, small cell carcinoma is most likely to

be associated with Cushing syndrome. Another clue is that surgical treatment is unlikely

to be curative; among lung cancers, this is particularly true of small cell carcinoma.

A, adenocarcinoma presents most commonly in the lung periphery and produces

malignant glands

B, malignant mesothelioma arises and preferentially involves the pleura, and is composed

of glandular elements and/or spindled cells.

C, non-Hodgkin lymphoma may morphologically resemble small cell carcinoma, but is

composed of dyscohesive cells and is not related to smoking. It is not associated with

ACTH secretion
E, squamous cell carcinoma may occur near the lung hilum, but is composed of large

cohesive cells that show keratinization, features not seen in small cell carcinoma. It is

typically associated with secretion of parathyroid hormone related peptide causing

hypercalcemia

Question #1
A 36-year-old woman has had increased malaise for 3 weeks and urine output <500 mL/day for the past
4 days. On examination, she has blood pressure 170/112 mm Hg and peripheral edema. Urinalysis shows
protein 1+ and blood 3+, but no glucose or ketones. Urine microscopic analysis shows RBCs and RBC
casts. Her serum urea nitrogen is 39 mg/dL, and creatinine is 4.3 mg/dL. Her serum complement C1q,
C3, and C4 are decreased. A renal biopsy is performed, and immunofluorescence microscopy shows this
pattern of staining with antibody to C3.

!
Which of the following mechanisms is most likely the cause of her renal disease?
A Deposition of anti-phospholipid antibodies on the basement membrane
B Deposition of anti-streptococcal antibodies that cross react with glomerular basement
membrane
C Deposition of complexes comprised of DNA and DNA antibodies
D Deposition of complexes comprised of histones and anti-histone antibodies
E Deposition of antibodies that cross react with alveolar basement membrane
Correct answer: C. The staining pattern is granular, characteristic of the deposition of immune complexes.
The clinical history and laboratory findings are strongly suggestive of lupus nephritis (renal disease
associated with SLE). The most common autoantibodies in these patients are anti-DNA antibodies.
A, anti-phospholipid antibodies are produced in SLE and cause coagulation abnormalities but they do not
deposit in the glomerulus.
B, anti-streptococcal antibodies form local immune complexes with streptococcal antigens deposited in
the GBM (post-streptococcal glomerulonephritis). Some anti-streptococcal antibodies also cross-react
with myocardial antigens and these cause rheumatic fever. In this case, the clinical history is not
consistent with post-streptococcal glomeruloneophritis.
D, in SLE, antibodies against numerous nucleoptroteins are more common than those reactive with
histones.
E, antibodies that bind to both glomerular and alveolar basement membranes are found in Goodpasture
syndrome. They deposit in the GBM in a characteristic linear pattern.

Question #2
A 7-year-old boy is recovering from impetigo. Physical examination shows five honey-colored crusts on
his face. The crusts are removed, and a culture of the lesions grows Streptococcus pyogenes. He is
treated with antibiotics. One week later, he develops malaise with nausea and a slight fever and passes
dark brown urine with red cell casts. Laboratory studies show a serum anti–streptolysin O titer of
1:1024. Which of the following is the most likely outcome of his renal disease?
A Chronic renal failure
B Complete recovery
C Acute renal failure
D Rheumatic heart disease
E Streptococcal urinary tract infection
Correct answer: B. The kidney disease is characteristic of an acute nephritic syndrome (microscopic
hematuria and systemic manifestations of acute inflammation). The streptococcal infection preceding the
renal disease and the presence of anti-streptococcal antibody are consistent with acute poststreptococcal
glomerulonephritis. The typical course of this disease in children is complete resolution in weeks or
months.
A, chronic renal failure would present with progressive loss of kidney function and not evidence of
acute inflammation or injury. It is a rare outcome of this disease in children as opposed to adults in who
one third develop chronic renal disease
C, the characteristic feature of acute renal failure is sudden loss of renal function manifested by
increased serum creatinine and often oliguria. It is a rare outcome in children with this disorder
D, rheumatic heart disease is also a sequel of streptococcal infection, but it affects the heart and not the
kidneys. The streptococci that cause rheumatic fever are of different subtypes than those that cause renal
disease. In addition, prompt antibiotic therapy usually prevents the development of rheumatic heart
disease (although not glomerulonephritis).
E, urinary tract infection manifests with local signs of inflammation (pain during urination, cloudy
urine with bacteria and leukocytes). Streptococcus is not a known cause of urinary tract infection.
Question #3
A 17-year-old girl living in the Congo has had a chronic febrile illness for 2 years. In the past 2 days she
notes her urine is smoky brown. On physical examination her blood pressure is 145/95 mm Hg.
Laboratory studies show her serum creatinine is 3.7 mg/dL, and urea nitrogen is 35 mg/dL. Urinalysis
shows 4+ blood with 1+ protein. A peripheral blood smear shows rare ring stage trophozoites. Light
microscopy shows hypercellular glomeruli. Immunofluorescence microscopy shows granular deposition
of IgG and C3 in glomerular capillary basement membranes. Electron microscopy shows electron-dense
subepithelial “humps.” Which of the following renal diseases is she most likely to have?
A Focal segmental glomerulonephritis
B Hereditary nephritis
C IgA nephropathy
D Lupus nephritis
E Acute post infectious glomerulonephritis
Correct answer: E The chronic febrile illness and the findings in the blood smear are diagnostic of
malaria; the Congo is an endemic area for this infection. In rare cases, patients with long-standing malaria
develop nephritis with microscopic hematuria and proteinuria and declining renal function. The lesions
are caused by deposition of immune complexes consisting of malarial antigens and anti-malarial
antibodies.
A, FSGS usually presents with the nephrotic syndrome, and immune complex deposition is not
present.
B, hereditary nephritis is caused by structural defects in the glomerular basement membrane and is not
an immune complex disease; it causes hematuria
C, IgA nephropathy is characterized by microscopic hematuria and IgA deposits but not immune
complexes.
D, the clinical presentation is not that of lupus, which has numerous systemic features
Question #4
A 45-year-old man has experienced increasing malaise, nausea, and reduced urine output for the past 3
days. On physical examination, he is afebrile and normotensive. Laboratory findings show a serum
creatinine level of 7.5 mg/dL. Urinalysis shows hematuria, but no pyuria or glucosuria. A renal biopsy
is done; the light microscopic picture is shown in the figure (Fig 20-10 PBD9).

Which of the following additional laboratory studies is most likely to be useful for classification and
treatment of this disease?
A Antinuclear antibody titer
B Anti–glomerular basement membrane antibody
C HIV-1 RNA copy level
D Quantitative serum immunoglobulins
E Rheumatoid factor
Correct answer: B. The acute presentation of oliguria and renal failure, and the histologic appearance
of crescentic glomerulonephritis, are characteristic of rapidly progressive glomerulonephritis (RPGN).
This clinical syndrome has numerous possible underlying causes, the most common of which is
Goodpasture syndrome (anti-GBM antibody syndrome), which is associated with the deposition of anti-
GBM antibody in the glomerulus in a linear immunofluorescence pattern.
A, anti-nuclear antibodies are seen in lupus and other autoimmune diseases. Lupus nephritis does not
present as crescentic GN with acute renal failure.
C, immune complex glomerulonephritis is a rare complication of HIV disease. It does not usually
present as crescentic GN.
D, measurements of total serum immunoglobulin would not be helpful for the diagnosis
E, serum rheumatoid factor is positive in rheumatoid arthritis and other similar autoimmune diseases.
When renal involvement occurs in these diseases, it does not usually take the form of crescentic GN.
Question #5
A 21-year-old previously healthy man has noticed blood in his urine for the past 2 days. He reports no
dysuria, frequency, or hesitancy of urination. On physical examination, there are no abnormal findings.
Laboratory findings show a serum urea nitrogen level of 39 mg/dL and creatinine level of 4.1 mg/dL. A
renal biopsy specimen is obtained; the immunofluorescence pattern of staining with antibody against
human IgG is shown below.

!
Which of the following therapeutic measures is most likely to be beneficial in this case?
A Plasma exchange
B Antibiotic administration
C Anti-TNF infusion
D Hepatitis B vaccine
E HIV antibody infusion
Correct answer: A. The acute onset of hematuria and evidence of renal failure suggests nephritis, and
the immunofluorescence staining pattern is indicative of deposition of anti-GBM antibody. This
individual most likely has a form of RPGN. In severe cases, removal of the antibody from the blood by
plasmapheresis is required to alleviate the acute renal failure. Chronic immunosuppression will be needed
to suppress the autoantibody production.
B, the lesion is not caused by an infection, and is not diagnostic of acute postinfectious GN, which is
characterized by immune complex deposition in a granular immunofluorescence pattern. Antibiotics
would not help even in postinfectious GN because the infection would have been in the past, and the
renal lesions are not caused by ongoing infection.
C, anti-TNF therapy is helpful for rheumatoid arthritis, inflammatory bowel disease, and other
autoimmune diseases in which T cell-mediated inflammation is prominent.
D, hepatitis vaccine is preventive, not therapeutic. Renal involvement is not common in hepatitis;
when it occurs, it is caused by deposition of immune complexes in a granular pattern.
E, infusion of HIV antibody has been used to treat rare cases of HIV infection but is not standard
therapy. HIV-associated nephropathy is not associated with linear deposition of anti-GBM antibody.
Question #6
A 44-year-old woman has developed a fever, nonproductive cough, and decreased urine output over
the past 3 days. On physical examination, her temperature is 37.7° C, and blood pressure is 145/95
mm Hg. She has had chronic sinusitis. On auscultation, crackles are heard over all lung fields. A chest
radiograph shows bilateral patchy infiltrates and nodules. The serum creatinine level is 4.1 mg/dL,
and the urea nitrogen level is 43 mg/dL. The results of serologic testing are negative for ANA, but
positive for C-ANCA (PR3-ANCA). A renal biopsy specimen shows glomerular crescents and
damage to small arteries. The result of immunofluorescence staining with anti-IgG and anti-C3
antibodies is negative. Which of the following additional microscopic findings is most likely to be
seen in this biopsy?
A Focal segmental glomerulosclerosis
B Glomerular basement membrane thickening
C Hyperplastic arteriolosclerosis
D Infiltration by T cells
E Necrotizing granulomatous vasculitis
Correct answer: E. Acute onset of renal failure suggests nephritis. The renal biopsy shows crescentic
glomerulonephritis (GN), a classical histologic manifestation of rapidly progressive GN (RPGN). One
of the causes of RPGN is vasculitis; the finding of small arterial damage supports this diagnosis. The
involvement of the lungs and sinuses is consistent with systemic necrotizing granulomatous vasculitis,
which often involves the kidneys. (It was previously called Wegener disease.)
A, FSGS typically presents with nephrotic syndrome and shows focal glomerular sclerosis, not
crescents.
B, GBM thickening is a nonspecific finding associated with several diseases, all of which present
with the nephrotic syndrome
C, hyperplastic arteriosclerosis is seen in malignant hypertension
D, T cell infiltrates are common in tubulointerstitial nephritis but are not common in vasculitis-
associated RPGN
Question #1

A previously healthy, 38-year-old woman has become increasingly obtunded in the past 4

days. On physical examination, she has scleral icterus, ascites, and asterixis. She is

afebrile, and her blood pressure is 110/55 mm Hg. Laboratory findings show a

prothrombin time of 38 seconds (INR 3.1), serum ALT of 1854 U/L, AST of 1621 U/L,

albumin of 1.8 g/dL, and total protein of 4.8 g/dL. Serum or blood levels of which of the

following will most likely be abnormal in this patient?

A Alkaline phosphatase

B Ammonia

C Amylase

D Anti-HCV

E Antinuclear antibody (ANA)

Correct answer: B. Jaundice, ascites, asterixis, and elevated AST and ALT, along with

coma all point to liver failure. One of the consequences of liver failure is the inability to

convert ammonia (produced in the intestine) into the nontoxic urea, which is excreted in

the urine. Increased ammonia levels correlate with encephalopathy; ammonia may

impair neuronal function.

A, alkaline phosphatase is produced in the liver and excreted in the bile. Elevated serum

levels suggest biliary obstruction (cholestasis).


C, amylase is a pancreatic enzyme whose levels are increased in acute pancreatitis because

of spillage from necrotic pancreatic acinar epithelial cells

D, anti-HCV antibodies would be positive in cases of hepatitis C, which usually does not

manifest as acute liver failure

E, ANA is elevated in patients with lupus and other autoimmune diseases.


Question #2

A 54-year-old woman has a long history of chronic hepatitis B infection and has had

increasing malaise for the past year. Physical examination now shows a firm nodular

liver, splenomegaly, and caput medusae. Laboratory findings show a serum albumin level

of 2.5 g/dL and prothrombin time of 28 seconds (normal ~11-13). Which of the following

complications is most likely to occur in this woman?

A Pulmonary edema

B Deep vein thrombosis of the legs

C Hematemesis

D Kernicterus

E Steatorrhea

Correct answer: C. The firm nodular liver indicates cirrhosis, in this case secondary to

chronic viral hepatitis. Splenomegaly and caput medusae (prominent veins around the

umbilicus) indicate portal hypertension. The laboratory findings indicate decreased

synthesis of albumin and clotting factors in the diseased liver. One of the complications

of portal hypertension is esophageal varices, which tend to bleed, especially in patients

with blood coagulation abnormalities.

A, pulmonary edema occurs in heart failure, not liver disease

B, Deep vein thrombosis, usually affecting the leg veins, result from prolonged immobility

due to any cause


D, kernicterus refers to severe hyperbilirubinemia in neonatal period – for example from

immune hemolysis- causes deposition of bilirubin in brain parenchyma, particularly in

basal ganglia; the bilirubin is toxic to the brain

E, steatorrhea is excessive fat in the stool (fat malabsorption), which is caused most often

by deficiency of pancreatic enzymes and celiac disease, not chronic liver disease
Question #3

A 57-year-old man has had increasing abdominal enlargement for 6 months. He has had an

episode of vomiting blood, 3 months before admission. On physical examination, there is

pedal edema, ascites, caput medusa, gynecomastia, and slight scleral icterus. Laboratory

testing shows serum albumin 2.1gm/dL, total proteins 5.4gm/dL; bilirubin 2.8g/dL, AST

450 U/L, and ALT 650U/L. The representative gross appearance of his liver is shown

below.

Fig 18-21A PBD9

Which of the following underlying diseases most commonly accounts for these findings?

A α1-Antitrypsin deficiency

B Chronic alcohol abuse

C Hepatitis E viral infection

D Hereditary hemochromatosis

E Primary biliary cholangitis

Correct answer: B. The clinical features (ascites, caput medusa, gynecomastia,

hematemesis and abnormal liver function tests) are diagnostic of cirrhosis with portal

hypertension, and liver failure. The diffuse nodularity with depressed scars are
characteristic of cirrhosis.. A common cause of this liver disease is chronic alcohol

abuse.

A, α1-antitrypsin deficiency causes pulmonary emphysema and occasionally chronic

hepatitis but is much less common than alcohol-induced liver disease

C, HEV is usually a self-limited infection, which causes chronic liver disease only in

immunosuppressed individuals

D, E, hemochromatosis and primary biliary cholangitis are rarer causes of cirrhosis. The

morphology is characteristic, with brown discoloration because of iron accumulation in

hemochromatosis and bile pigmentation in PBC.


Question #4

A 65-year-old man with a history of alcohol abuse has had hematemesis for the past day.

Physical examination reveals mild jaundice, spider angiomas, and gynecomastia. He has

mild pedal edema, normal jugular venous pressure (JVP), and a massively distended

abdomen. Paracentesis is performed and the fluid obtained shows accumulation of

protein-poor fluid that is free of inflammatory cells. Which of the following factors is

most likely to be responsible for the collection of abdominal fluid in this man?

A Congestive heart failure

B Hepatorenal syndrome

C Hyperbilirubinemia

D Portosystemic shunts

E Portal hypertension

Correct answer: E. The history of alcohol abuse suggests liver disease and hematemesis

indicates portal hypertension. Impaired ability to catabolize estrogens may cause

symptoms of estrogen excess such as hypogonadism and gynecomastia in males and

spider angiomas; mild pedal edema is because of reduced plasma albumin. The likely

cause of fluid-filled abdominal distention (ascites) is portal hypertension; the transudate

of protein-poor fluid is consistent. Ascites may also occur in congestive heart failure, but

in this case the jugular venous pressure would be elevated.

A, unlikely because of normal jugular venous pressure


B, hepatorenal syndrome refers to kidney abnormalities secondary to liver disease. It

would not cause the manifestations of portal hypertension seen in this patient.

C, hyperbilirubinemia secondary to cholestasis occurs often in liver disease, but it does not

cause ascites

D, portosystemic shunts may occur in severe portal hypertension, they cause esophageal

varices, caput medusae, and hemorrhoids but are not the cause of ascites.
Question #5

A 27-year-old man develops malaise, fatigue, and loss of appetite three weeks after a meal

at the Trucker’s Cafe. He notes passing dark urine. On physical examination, he has mild

scleral icterus and right upper quadrant tenderness. Laboratory studies show serum AST

of 62 U/L and ALT of 58 U/L. The total bilirubin concentration is 3.9 mg/dL His

symptoms abate over the next 3 weeks. On returning to the cafe, he finds that the city’s

health department has closed it. Which of the following serologic test results is most

likely to be positive in this patient?

A Anti-HAV

B Anti-HBc

C Anti-HBs

D Anti-HCV

E Anti-HDV

Correct answer: A. The symptoms are consistent with acute hepatitis – dark urine and

icterus (jaundice) are because of cholestasis and increased bilirubin in the blood; serum

transaminases are elevated, indicating liver injury. The association with food

consumption in a café that has been closed by health authorities is suggestive of an

infection acquired from contaminated food or water, most likely hepatitis A virus in this

case.
B, C, D, hepatitis B and C are acquired by parenteral inoculation (contaminated needles,

blood transfusions in the past) and sexual transmission, or perinatal infection in children.

Both are capable of establishing chronic hepatitis

E, hepatitis D is closely associated with HBV and is usually a co-infection

8 An 18-year-old man comes to his physician for a routine health maintenance examination.

On physical examination, there is no left testis palpable in the scrotum. The patient is

healthy, has had no major illnesses, and has normal sexual function. This person is at a

higher risk for which of the following lesions?

A Testicular germ cell tumor

B Tubercular epididymitis

C Syphilis

D Mumps infection

E Squamous cell carcinoma of the scrotal skin

Answer, A Testicular germ cell tumor, which occurs at higher frequency in individuals

with cryptorchidism (non-descended testis); the risk is 3-5 fold higher

B,C, D, E Tubercular epididymitis syphilis, mumps, and squamous cell carcinoma of

scrotal skin are all unrelated to cryptorchidism

9 A 29-year-old man complains of a vague feeling of painless heaviness in the scrotum for

the past 5 months. Physical examination shows that the right testis is slightly larger than
the left testis. An ultrasound scan shows a solid, circumscribed, 1.5-cm mass in the body

of the right testis. The representative gross appearance of the mass is shown in the figure

(Fig 18.3 BP10). A biopsy is done, and microscopic examination of the mass shows

uniform nests of cells with distinct cell borders and clear cytoplasm. There are aggregates

of lymphocytes between these nests of cells. Which of the following features is most

characteristic of this lesion?

A Isochromosome 12

B 46,XXY karyotype

C Presence of HPV 16 or 18 in the nuclei

D Synthesis of human chorionic gonadotropin

E Secretion of α-fetoprotein

Answer, A A painless solid mass in the testis is highly suspicious of a testicular tumor.

The microscopic appearance is typical of a seminoma. Isochromosome 12, is present

in a high fraction of seminomas.

B 46,XXY karyotype is diagnostic of Klinefelter syndrome and is not a feature of

seminoma. Klinefelter syndrome is associated with testicular atrophy

C Presence of high risk HPV 16 or 18 in the nuclei is associated with carcinoma of the

cervix but seminoma.

D Synthesis of human chorionic gonadotropin (HCG) is a feature of germ cell tumors

with syncytiotrophoblasts, particularly choriocarcinoma. HCG may occasionally be

mildly elevated in some seminomas that have syncytial cells, but this is not a major

feature of this particular germ cell tumor.


E Secretion of α-fetoprotein is a feature, among germ cell tumors, of yolk sac tumor,

and is not a feature of seminoma.

10 A 28-year-old man has noticed increasing enlargement and a feeling of heaviness in his

scrotum for the past year. On physical examination, the right testis is twice its normal size

and is firm but not tender. An ultrasound examination shows a 3.5-cm solid right

testicular mass. Abdominal CT scan shows enlargement of the para-aortic lymph nodes.

Multiple lung nodules are seen on a chest radiograph. Laboratory findings include

markedly increased serum levels of chorionic gonadotropin and α-fetoprotein. Which of

the following neoplasms is most likely present?

A Choriocarcinoma

B Large diffuse B-cell lymphoma

C Pure embryonal cell carcinoma

D Pure seminoma

E Mixed germ cell tumor

Answer, E Mixed germ cell tumor with areas of both choriocarcinoma and yolk sac

tumor, associated with secretion of human chorionic gonadotropin (HCG) and α-

fetoprotein, respectively. Choriocarcinomas are known to spread to lungs via blood

stream and several testicular tumors including seminoma spread to para-aortic lymph

nodes.

A Choriocarcinoma would be associated with secretion of human chorionic

gonadotropin (HCG) only.


B Large diffuse B-cell lymphoma occurs rarely in the testis but is not associated with

secretion of human chorionic gonadotropin (HCG) or α-fetoprotein

C Pure embryonal cell carcinoma would not be associated with secretion of human

chorionic gonadotropin (HCG) or α-fetoprotein

D Pure seminoma would not be associated with secretion of human chorionic

gonadotropin (HCG) or α-fetoprotein

11 A 32-year-old man has noticed an increased feeling of heaviness in his scrotum for the

past 10 months. On physical examination, the left testis is three times the size of the right

testis and is firm on palpation. An ultrasound scan shows a 6-cm solid mass within the

body of the left testis. Laboratory studies include an elevated serum α-fetoprotein level.

Which of the following cellular components is most likely to be present in this mass?

A Cytotrophoblasts

B Embryonal carcinoma cells

C Syncytiotrophoblasts

D Seminoma cells

E Yolk sac cells

Answer, E Yolk sac tumor is associated with secretion of α-fetoprotein.

A Cytotrophoblasts are a feature of choriocarcinoma and do not secrete α-fetoprotein.

B Embryonal carcinoma cells do not secrete α-fetoprotein.

C Syncytiotrophoblasts are a feature of choriocarcinoma and are associated with human

chorionic gonadotropin (HCG) secretion, not α-fetoprotein secretion.


D Seminoma cells do not secrete α-fetoprotein.

12 A 26-year-old man has occasionally felt pain in the scrotum for the past 3 months. On

physical examination, the right testis is tender, but does not appear to be enlarged. An

ultrasound scan shows a 1.5-cm mass within the right testis. A right orchiectomy is

performed, and gross examination shows the mass to be hemorrhagic and soft.

Microscopic examination shows that areas of viable tumor are composed of cuboidal

cells intermingled with large eosinophilic syncytial cells containing multiple dark,

pleomorphic nuclei. The syncytial cells are most likely to produce the following?

A α-Fetoprotein

B Carcinoembryonic antigen

C Estrogens

D Human chorionic gonadotropin

E Testosterone

Answer, D Human chorionic gonadotropin. The description is consistent with the

presence of both cytotrophoblasts and syncytiotrophoblasts, features of

choriocarcinoma, a tumor that secretes HCG in large amounts.

A α-Fetoprotein is made by embryonal carcinoma and hepatocellular carcinoma.

B Carcinoembryonic antigen is mainly made by various carcinomas, including colonic

carcinoma.

C Estrogens are made by certain ovarian tumors, particularly granulosa cell tumors.

E Testosterone is made by certain testicular tumors, particularly Leydig cell tumors.


5 A healthy 30-year-old woman comes to the physician for a routine health maintenance

examination. No abnormalities are found on physical examination. A screening Pap

smear shows cells consistent with a low-grade squamous intraepithelial lesion (LSIL).

Subsequent cervical biopsy specimens confirm the presence of cervical intraepithelial

neoplasia (CIN) I. Which of the following risk factors is most likely related to her Pap

smear findings?

A Diethylstilbestrol (DES) exposure

B Multiple sexual partners

C Oral contraceptive use

D Prior treatment for a malignancy

E Smoking

Answer,

B, Multiple sexual partners increases the probability of infection with high risk HPV,

the cause of cervical squamous intraepithelial lesions (SIL).

A, Diethylstilbestrol (DES) exposure is associated with a rare tumor called clear cell

carcinoma of the vagina, not SIL.

C, Oral contraceptive use does not increase the risk for SIL.

D, Prior treatment for a malignancy does not increase the risk for SIL. Chemotherapy

for treatment of certain malignant tumors is sometimes followed by leukemia.

E, Smoking does not increase the risk for SIL.


7 A 40-year-old woman has a Pap smear as part of a routine health maintenance

examination. There are no remarkable findings on physical examination. The Pap smear

shows cells consistent with a high-grade squamous intraepithelial lesion (HSIL) with

human papillomavirus type 18. Cervical biopsy specimens are obtained, and microscopic

examination confirms the presence of extensive dysplasia (CIN III) along with intense

chronic inflammation with squamous metaplasia in the endocervical canal. What is the

rationale for recommending surgical excision for this patient?

A Her reproductive years are over

B Risk for bleeding

C Premenopausal state

D Presence of chronic cervicitis

E Risk for invasive carcinoma

Answer, E, Risk for invasive carcinoma is high in the women with a high grade

precancer. Removal of the lesion will prevent progression to cervical carcinoma.

A, Conization (removal of the region of the lesion) is compatible with subsequent

pregnancy and childbirth, albeit with an increased risk of complications related to

cervical incompetence.

B, HSIL usually is not associated with bleeding.

C, Premenopausal state is not a consideration when treating HSIL.

D, Chronic cervicitis is a non-specific finding and is almost never treated.


8 A 34-year-old woman who has been using oral contraceptives for 10 years has a routine

Pap smear for the first time. The results indicate that dysplastic cells are present,

consistent with a high-grade squamous intraepithelial lesion (HSIL). Surgical excision

shows cervical intraepithelial neoplasia (CIN) III. What is the most likely molecular

pathogenesis for this finding?

A Estrogenic stimulation of cell proliferation

B Inheritance of a tumor suppressor gene mutation

C Recurrent gonococcal cervicitis

D Up-regulation of BCL2 gene

E Viral inactivation of the RB1 gene product

Answer, E. The RB1 gene product, RB, is inactivated by the E7 protein encoded in the

genome of high-risk strains of HPV.

A, oral contraceptive use is not associated with HSIL.

B, HSIL and cervical carcinoma are not among the various cancers that are highly

associated with inheritance of a defective copy of a tumor suppressor gene.

C, gonococcal cervicitis is not related to HSIL.

D, up-regulation of BCL2 gene expression is not related to HSIL.

9 A 13-year-old girl began menstruation 1 year ago. She now has abnormal uterine bleeding,

with menstrual periods that are 2 to 7 days long and 2 to 6 weeks apart. The amount of

bleeding varies from minimal spotting to a very heavy flow. On physical examination,
there are no remarkable findings. A pelvic ultrasound scan shows no abnormalities.

Which of the following is most likely to produce these findings?

A Anovulatory cycles

B Ectopic pregnancy

C Endometrial carcinoma

D HPV infection

E Endometrial hyperplasia

Answer, A. During puberty ovulation may be irregular, leading to anovulatory cycles

with little or no shedding of the endometrial lining, followed by ovulatory cycles

during which excess shedding occurs.

B, Ectopic pregnancy presents with pain and sometimes massive bleeding, leading to

shock.

C, Endometrial carcinoma is a disease of middle-aged adult women.

D, HPV infection is sexually transmitted and would present with genital warts or

potentially cervical neoplasia; the latter occurs years after initial infection.

E, Endometrial hyperplasia is a disorder mainly seen in premenopausal or

perimenopausal women with hyperestrinism, sometimes due to obesity.

10 A 35-year-old woman presents with infertility. She has had dysmenorrhea, dyspareunia,

and pelvic pain on defecation for 4 years. Laparoscopic examination reveals red-blue

nodules on the surface of the uterus and extensive adhesions between ovaries and the

fallopian tubes. Histologic examination of a biopsy from one of the nodules shows
hyperplastic endometrial glands and hemorrhage in the stroma. Molecular analysis fails

to reveal mutations in PTEN, TP53 or mismatch repair genes. Which of the following is

the most appropriate treatment modality in this case?

A Estrogen inhibitors

B Chemotherapy

C Antibiotics

D Insulin

E Oophorectomy

Answer, A, Estrogen inhibitors. The findings given are consistent with endometriosis,

the presence of ectopic endometrial glands and stroma. Compared to normal

endometrium, endometriotic tissue expresses higher levels of inflammatory mediators

because factors made by stromal cells recruit and activate macrophages. Stromal cells

also make aromatase leading to local production of estrogens. Hence anti-

inflammatory agents and estrogen inhibitors are used to treat endometriosis.

B, Chemotherapy is not indicated in this benign condition.

C, Since endometriosis is not caused by infections, antibiotics are not useful

D, Insulin is used to treat diabetes which is not a risk factor for endometriosis

E, Oophorectomy is not necessary, since hormone levels can be manipulated medically.

11 A 42-year-old woman has had menometrorrhagia for the past 2 months. Her BMI is 32.

She has no history of prior irregular menstrual bleeding, and she has not yet reached

menopause. On physical examination, there are no vaginal or cervical lesions, and the
uterus appears normal in size, but there is a right adnexal mass. An abdominal ultrasound

scan shows the presence of a 7-cm solid right adnexal mass. Endometrial biopsy shows

hyperplastic endometrium, but no cellular atypia. What is the most likely lesion that

underlies her menstrual abnormalities?

A Corpus luteum dysfunction

B Endometriosis

C Granulosa cell tumor

D Mature cystic teratoma

E Metastasis

Answer, C, Granulosa cell tumor. The size of the mass is consistent with a neoplasm,

and the presence of endometrial hyperplasia suggests hyperestrinism, Granulosa cell

tumors make estrogens

A, Corpus luteum dysfunction would not be associated with a 7 cm solid mass. Corpus

luteum cysts do not produce estrogens

B, Endometriosis usually produces cystic lesions associated with pain. Endomteriosis

affecting the ovary can produce a mass but endometrial tissue does not produce

estrogens

D, Mature cystic teratoma is a cystic lesion. Teratomas do not contain estrogen

producing tissue.

E, Metastasis is unlikely in the absence of other symptoms and, when present it does not

produce estrogens.
12 A 62-year-old childless woman noticed a blood-tinged vaginal discharge twice during the

past month. Her last menstrual period was 10 years ago. Bimanual pelvic examination

shows that the uterus is normal in size, with no palpable adnexal masses. There are no

cervical erosions or masses. Her body mass index is 33. Her medical history indicates

that for the past 30 years she has had hypertension and type 2 diabetes mellitus. An

endometrial biopsy specimen is most likely to show which of the following?

A Endometrial carcinoma

B Choriocarcinoma

C Leiomyosarcoma

D Endometritis

E Squamous cell carcinoma

Answer, A, Endometrial carcinoma, which produces bleeding and an increase in uterine

size, is likely in this post-menopausal woman. She has multiple risk factors for

endometrial cancer: obesity, hypertension and diabetes.

B, Choriocarcinoma occurs in the setting of pregnancy or the immediate post-partum

setting.

C, Yterine leiomyosarcoma is much rarer than endometrial carcinoma and is not

associated with obesity, hypertension, and diabetes.

D, Endometritis would not increase the size of the uterus and is not associated with the

listed risk factors

E, Squamous cell carcinoma is very rare in the uterus.


Question #2

A 41-year-old woman notices that her gloves from the previous winter no longer fit her

hands. Her facial features have become coarse in the past year, and her voice seems

deeper. On physical examination, her blood pressure is 140/90 mm Hg. Laboratory

studies indicate a fasting serum glucose level of 138 mg/dL and hemoglobin A1c level of

8.6%. Which of the following additional test results is most likely to indicate the cause of

her physical and laboratory findings?

A Abnormal glucose tolerance test result

B Failure of oral load of glucose to suppress growth hormone levels

C Hyperprolactinemia

D Increased serum TSH level

E Increased serum catecholamine levels

Correct answer: B. The physical findings suggest excess of growth hormone, and the

laboratory evidence of diabetes is consistent with a growth-hormone producing

(somatotroph) pituitary adenoma. Growth hormone induces peripheral insulin resistance.

In normal individuals, growth hormone production is suppressed by insulin produced in

response to an oral glucose load (an example of feedback regulation), but adenomas are

autonomous and do not respond to normal regulatory signals.


A, an abnormal glucose tolerance test is likely in this patient. It indicates diabetes mellitus

but not the cause of diabetes

C, excessive prolactin secretion is seen with prolactin-producing (lactotroph) adenoma, and

presents with galactorrhea, amenorrhea and infertility

D, increased serum TSH is seen with TSH-producing (thyrotroph) adenoma, and presents

with signs and symptoms of hyperthyroidism

E, increased serum catecholamine is seen with pheochromocytoma, a tumor of the adrenal

medulla and other neuroendocrine glands


Question #3

A 21-year-old woman delivers a term infant after an uncomplicated pregnancy. The placenta

cannot be delivered, however, and there is substantial hemorrhage, requiring transfusion

of 10 U of packed RBCs. Over the next 3 months, she is unable to produce sufficient

milk to breastfeed her infant, and she becomes increasingly fatigued. Laboratory studies

show hyponatremia and hyperkalemia, normal serum calcium and phosphate, and normal

blood sugar. Over the next 5 months, her menstrual cycles do not return. Which of the

following laboratory findings is now most likely to be reported in this woman?

A Decreased corticotropin-releasing hormone

B Decreased oxytocin

C Failure of antidiuretic hormone release

D Decreased growth hormone

E Increased corticotropin

Correct answer: D. Ischemic necrosis of the pituitary (Sheehan syndrome) is a rare post-

partum complication. The anterior pituitary enlarges during pregnancy due to an

increased demand for prolactin. The blood supply does not increase proportionately,

making the gland susceptible to ischemia. Ischemic necrosis of the pituitary is most likely

to occur in women who develop hypotension during delivery. The anterior pituitary is

much more likely to be affected than posterior pituitary. This patient shows all the

features of hypopituitarism: inability to breast feed due to lack of prolactin; loss of


menstrual cycles due to deficiency of FSH and LH; hyponatremia and hyperkalemia due

to adrenal insufficiency resulting from reduced ACTH production

A, corticotropin-releasing hormone level will be normal because hypothalamus is not


affected

B, C, oxytocin and ADH will be unaffected because they are produced by posterior pituitary

E. Corticotropin (ACTH) would be reduced because it is produced in anterior pituitary


Question #4

A 42-year-old man has had polyuria and polydipsia for the past 4 months. His medical

history shows that he fell off a ladder and hit his head just before the onset of these

problems. On physical examination, there are no specific findings. Laboratory findings

include serum Na+, 155 mmol/L; K+, 3.9 mmol/L; Cl–, 111 mmol/L; and osmolality, 350

mOsm/mL. The urine specific gravity is 1.002. This patient is most likely to have a

deficiency of which of the following hormones?

A Corticotropin

B Thyrotropin

C Oxytocin

D Prolactin

E Vasopressin

Correct answer: E. The clinical history, increased serum Na+ and osmolality, and decreased

urine specific gravity are diagnostic of diabetes insipidus, in which secretion of

antidiuretic hormone (ADH, also called vasopressin) is reduced. The posterior pituitary

may be damaged by head trauma, tumors, or surgical procedures involving the

hypothalamus or pituitary. The result of ADH deficiency is inadequate resorption of water

in the kidney, producing polyuria. The excessive renal water loss elevates the sodium

concentration and osmolality in the blood, inducing thirst and frequent drinking of water

(polydipsia).

A, B, D, deficiency of these trophic hormones would be seen in anterior pituitary lesions


C, oxytocin deficiency is seen in posterior pituitary or hypothalamic diseases; it usually

manifests with problems during childbirth and lactation.


Question #5

A 69-year-old man has become progressively obtunded over the past week. He has an 80

pack-year history of smoking cigarettes. On physical examination, he is afebrile and

normotensive. A head CT scan shows no intracerebral hemorrhages. Laboratory findings

include serum Na+ of 115 mmol/L, K+ of 4.2 mmol/L, Cl– of 85 mmol/L, and bicarbonate

of 23 mmol/L. The serum glucose is 80 mg/dL, urea nitrogen is 19 mg/dL, and creatinine

is 1.7 mg/dL. Which of the following neoplasms is most likely to be present in this man?

A Adenohypophyseal adenoma

B Adrenocortical carcinoma

C Pheochromocytoma

D Small cell lung carcinoma

E Renal cell carcinoma

Correct answer: D. The clinical history of neurologic dysfunction and the laboratory finding

of hyponatremia are suggestive of inappropriate ADH secretion. This is often seen in

small cell lung cancer, when the tumor cells secrete the hormone (an example of a

paraneoplastic syndrome). The history of smoking is consistent with lung cancer.

A, anterior pituitary adenoma would not secrete ADH, which is a posterior pituitary

hormone

B, C, adrenal tumors also do not produce ADH

E, paraneoplastic syndromes associated with excessive hormone secretion occur with renal

cell carcinoma, but the hormone is usually erythropoietin not ADH.


Question #6

A 47-year-old woman has had increasing fatigue with dyspnea and reduced exercise

tolerance for the past year. She also complains of constipation. On examination she has

nonpitting edema of the lower extremities. Laboratory studies show a serum TSH level of

10 mU/L and T4 level of 2 µg/dL. She is most likely to have pathologic findings affecting

which of the following cells?

A Thyrotrophs

B Hypothalamic cells

C Somatotrophs

D Thyroid C cells

E Thyroid follicular cells

Correct answer: E. The clinical features suggest primary hypothyroidism, and the laboratory

findings of reduced T4 and increased TSH are diagnostic. Primary hypothyroidism may

be caused by autoimmune thyroid disease (Hashimoto thyroiditis), dietary iodine

deficiency or it may be iatrogenic (post surgery for cancer). Whatever the underlying

cause, the thyroid follicular cells are not functioning normally.

A, B, thyrotroph or hypothalamic dysfunction as a cause of hypothyroidism would show

reduced TSH

C, somatotroph abnormalities would not affect thyroid hormones

D, thyroid C cells are neuroendocrine cells that produce calcitonin not thyroid hormone
Question #7

A 43-year-old woman has had increasing lethargy and weakness over the past 3 years. She

has cold intolerance and wears a sweater in the summer. One year ago, she had

menorrhagia, but now she has oligomenorrhea. She has difficulty concentrating, and her

memory is poor. She has chronic constipation. On physical examination, her temperature

is 35.5° C, pulse is 54/min, respirations are 13/min, and blood pressure is 110/70 mm Hg.

Laboratory findings show hemoglobin, 13.8 g/dL; hematocrit, 41.5%; glucose, 73 mg/dL;

and creatinine, 1.1 mg/dL. The representative microscopic appearance of her causative

disease is shown below.

Fig 20.9 BP10

Which of the following serologic test findings is most likely to be positive in this woman?

A Anticentromere antibody

B Anti–double-stranded DNA antibody

CC Antimitochondrial antibody

DD Antithyrotroph antibody

EE Anti–thyroglobulin antibody
Correct answer: EE. The clinical history of lethargy, cold intolerance and oligomenorrhea

and the physical findings of low body temperature and low heart and respiratory rates

are suggestive of hypothyroidism. The lymphoid follicles and large pink nodules of

Hurthle cells seen microscopically are diagnostic for Hashimoto thyroiditis. (An episode

of menorrhagia a year ago may indicate transient hyperthyroidism, which is often seen

in these patients.) The disease is caused by multiple autoantibodies against thyroid

follicular antigens.

A, anticentromere antibodies (one type of antinuclear autoantibody) are most often

associated with limited systemic sclerosis

B, anti-dsDNA antibody is classic for SLE

C, antimitochondrial antibody is associated with primary sclerosing cholangitis

D, hypothyroidism in this case is primary (due to thyroid abnormality) and not because of

lesions of thyrotrophs in the anterior pituitary

Question #8

A 49-year-old woman has had a severe headache for 2 days. On physical examination, she is

afebrile and normotensive. Funduscopic examination shows papilledema on the right.

One day later, she has right pupillary dilation and impaired ocular movement. She then

becomes obtunded. Which of the following lesions best explains these findings?

A Chronic subdural hematoma

B Temporal lobe herniation


C Non communicating hydrocephalus

D Hydrocephalus ex vacuo

E Ruptured middle cerebral berry aneurysm

Correct answer: B. Papilledema indicates increased intracranial pressure and the headache

suggests a cerebral hemorrhage as the underlying cause. Dilated pupils and obtundation

suggest a transtentorial temporal lobe herniation secondary to the increased intracranial

pressure – pupillary dilation is because of compression of the third cranial nerve and

obtundation and coma because of compression of the midbrain.

A, chronic subdural hematoma develops following trauma and is insidious in onset and

progresses slowly

C, noncommunicating hydrocephalus is a localized expansion of one of the ventricles,

usually secondary to a block. The presentation is usually subtle, and reflects the primary

disease.

D, hydrocephalus ex vacuo is a compensatory increase in the volume of the ventricle

secondary to loss of brain parenchyma from any cause (e.g. infarct). It is of no clinical

consequence.

E, a ruptured aneurysm causes subarachnoid hemorrhage, and presents with sudden severe

headache and often rapid loss of consciousness.


Question #9

A 21-year-old woman incurs a blow to her head from a fall while mountain biking. She then

has loss of consciousness for 5 minutes. On examination her deep tendon reflexes are

diminished. A head CT scan 6 hours later shows no abnormalities. She recovers over the

next week, with no neurologic deficits, but cannot remember this event. Which of the

following is the most likely consequence of her injury?

A Arteriolosclerosis

B Concussion

C Hydrocephalus

D Post-traumatic dementia

E Diffuse axonal injury

Correct answer: B. Transient loss of consciousness and absent reflexes following brain

trauma are characteristic of concussion. Recovery, with persisting amnesia is also

typical.

A, arteriolosclerosis is narrowing of small vessels, not a consequence of her injury

C, hydrocephalus is a slowly developing increase in CSF volume

D, post-traumatic dementia is a neurodegenerative disorder caused by repeated brain trauma

(now called chronic traumatic encephalopathy)

E, diffuse axonal injury is seen in global cerebral ischemia, usually resulting from severe

hypotension
Question #10

An 83-year-old woman slips in the bathtub in her home and falls backward, striking her

head. She is taken to the emergency department, where examination shows a 3-cm

reddish, slightly swollen area over the occiput. She is arousable but somnolent. There are

no motor or sensory deficits. There is no papilledema. CT scan of the head is performed.

Acute hemorrhage in which of the following locations is most likely to be seen?

A Basal ganglia

B Cerebral ventricle

C Epidural space

D Inferior frontal lobe

E Occipital lobes

Answer: D, this is classic contrecoup injury, in which the moving head strikes an object and the
force is transmitted to the opposite side of the head. A fall backwards is most likely to injure
inferior frontal lobes. A blow to a stationary head is more likely to produce coup injury at the site
of the blow.

A, B, C and E would not be affected by this type of fall.


Question #11

A 19-year-old snowboarder wearing protective equipment consisting of a baseball cap,

baggy shorts, and a flak jacket flew off a jump and hit a tree. He was initially

unconscious, and then “came to” and wanted to try another run, but his friends thought it

best to call for help. On the way to the emergency department, he became comatose.

Physical examination now shows left papilledema. Skull radiographs show a linear

fracture of the left temporoparietal region. This clinical picture is most consistent with

which of the following lesions?

A Acute leptomeningitis

B Contusion of frontal lobes

C Middle meningeal artery laceration

D Ruptured berry aneurysm

E Tearing of cerebral bridging veins

Correct answer: C. A complication of traumatic skull fracture is an epidural hemorrhage

secondary to tearing of a meningeal artery. A lucid interval followed by rapid

neurological deterioration is typical of this type of trauma. Papilledema indicates

increased intracranial pressure.

A, acute meningitis develops more slowly with signs of inflammation (headache, neck

rigidity), not associated with trauma or skull fracture

B, contusions are small areas of hemorrhage with tissue injury, which may be associated

with temporary loss of consciousness (concussion)


D, a ruptured aneurysm causes subarachnoid hemorrhage, and presents with sudden severe

headache and often rapid loss of consciousness

E, tearing of bridging veins leads to subdural hematoma, the symptoms evolve more slowly

and often consist of headache and confusion


Question #12

An 80-year-old resident of a nursing home is admitted to the hospital because of recent

onset of fluctuating levels of consciousness with headache and confusion for the past 2

days. On physical examination, she is arousable, but disoriented and irritable. Vital signs

include temperature of 36.9° C and blood pressure of 130/85 mm Hg. There is

papilledema on the right. CT scan of the head shows a collection of blood in the subdural

space on the right. Which of the following vascular lesions most likely produced these

findings?

A Bleeding from an arteriovenous malformation

B Laceration of the middle meningeal artery

C Rupture of a saccular aneurysm

D Tearing of the cerebral bridging veins

E Thrombosis of the middle cerebral artery

Correct answer: D. Subdural hemorrhage is caused by tearing of bridging veins and is

usually secondary to trauma. The gradual development of symptoms and evidence of

increased intracranial pressure (papilledema) are characteristic.

A, bleeding from an AVM results in an intracerebral or subarachnoid hemorrhage

B, laceration of a meningeal artery causes an epidural hemorrhage

C, rupture of an aneurysm causes a subarachnoid hemorrhage

E, thrombosis of the middle cerebral artery causes cerebral infraction, which could be

hemorrhagic but the blood, does not get into subdural space.
Question #13

A 68-year-old woman with atrial fibrillation suddenly lost consciousness and fell to the

ground. When she became arousable, she was unable to move her left arm and had

difficulty speaking. On physical examination, her temperature was 37° C, pulse was 81/

min, respirations were 18/min, and blood pressure was 135/85 mm Hg. Despite optimal

medical therapy, she did not regain consciousness and the family elected to withdraw

care, The figure shows the representative gross appearance of her brain at autopsy.

Fig 23-7 A BP10

The development of such a lesion most likely resulted from which of the following

conditions?

A Arteriovenous malformation

B Embolic arterial occlusion

C Metastatic carcinoma

D Organizing subdural hematoma

E Cerebral amyloid angiopathy


Correct answer: B. A focal hemorrhagic infarct (area of necrosis with multiple small

hemorrhages in a particular arterial distribution) is most often caused by an embolus.

Atrial fibrillation predisposes to the formation of mural thrombi in the atrium, a likely

source of the embolus in this case. Speech difficulty and paralysis of the left arm are

typical of infarcts affecting the right cerebral hemisphere.

A, an AVM can rupture and cause intracerebral hemorrhage but not infarcts

C, metastatic carcinoma would usually be a more circumscribed lesion and would not

present with the symptoms in this case

D, the lesion is intracerebral, not subdural

E, cerebral amyloid angiopathy is characterized by amyloid deposits in cerebral arteries.

The deposits weaken the vessel wall and may lead to multiple lobar hemorrhages in the

cerebral cortex.

Question #11

A 30-year-old woman has experienced pain in the area of the left knee for 1 month. On

physical examination, there is tenderness to palpation of the distal left thigh and knee.

The area is firm, but there is no erythema or warmth. A radiograph of the left leg shows a

7-cm mass in the distal femoral epiphyseal area, with a “soap bubble” appearance.

Microscopic examination of a curettage specimen of the lesion shows the findings in the

figure.
Fig 21-26 BP10

The lesion recurs in the next year; it is excised and does not recur again. The formation of

multinucleated cells in this lesion is most likely due to the production of which of the

following factors?

A Osteoprotegerin

B IL-6

C RANK Ligand

D VEGF

E Fibroblast growth factor

Correct answer: C. This solitary lytic bone tumor, occurring in epiphysis, and containing

abundant multinucleate giant cells along with a smaller number of neoplastic stromal

cells is a giant cell tumor. The multinucleate cells are derived from osteoclasts and are

responsible for bone destruction. In some of these tumors, the stromal cells produce high

levels of RANK-ligand, a cytokine that promotes osteoclast differentiation and

activation.

A, osteprotegerin is a decoy ligand that inhibits the binding of RANK-ligand to its receptor

and thus reduces osteoclast activity


B, IL-6 also activates osteoclasts but is not as potent and is not associated with giant cell

tumor

D, E, VEGF and FGF are growth factors for endothelial cells and fibroblasts and do not

activate osteoclasts
Question #12

A 45-year-old man has experienced pain in the area of the left hip and upper thigh for the

past 7 months. On physical examination, there is tenderness on deep palpation of the left

side of the groin. The range of motion at the left hip is decreased, but there is no swelling

or warmth on palpation. Pelvic and left leg radiographs show a neoplastic mass arising in

the ilium. Microscopically, the mass shows cells embedded in a myxoid stroma. The

tumor cells harbor mutations in IDH1 and IDH2 genes. Which of the following cell types

is most likely to be proliferating in this mass?

A Chondrocyte

B Osteoblast

C Osteoclast

D Plasma cell

E Primitive neuroectodermal cell

Answer: A, This is a chondrosarcoma; unlike osteosarcomas, these tumors arise from

the axial skeleton and most often involve the pelvis ( as in this case), shoulder and

ribs. Approximately 50% harbor mutations in IDH1 and IDH 2 genes which provide

targets for therapy

B, osteoblasts proliferate in osteosarcomas

C, osteoclasts are prominent in giant cell tumors

D, plasma cells proliferate in multiple myeloma

E, primitive neuroectodermal cells proliferate in Ewing tumor


Question #13

A 13-year-old, previously healthy boy has had pain in the right leg for the past month. There

is no history of trauma or recent illness. On physical examination, there is warmth and

tenderness to palpation of the right lower thigh anteriorly, and the circumference of the

right thigh is slightly larger than that of the left. His temperature is 39° C. A radiograph of

the right leg shows a 6-cm expansile mass in the diaphyseal region of the right lower

femur that extends into the soft tissue and is covered by layers of reactive bone. A biopsy

of the mass is done, and microscopic examination shows sheets of closely packed

primitive cells with small, uniform nuclei and only scant cytoplasm.

Fig 21-24 BP10

Karyotypic analysis of the tumor cells shows a t(11;22) translocation. What is the most

likely diagnosis?

A Chondrosarcoma

B Ewing sarcoma

C Giant cell tumor

D Osteosarcoma

E Plasmacytoma
Correct answer: B. The diaphysial location and occurrence in a child suggests Ewing

sarcoma, and the histologic appearance (small round blue cells) and characteristic

chromosomal translocation are diagnostic. The translocation fuses the Ewing sarcoma

gene EWSR1 to a gene encoding a transcription FLI1, but how this causes the tumor is

unknown. The other tumors in the choices have distinct histology and do not have this

translocation.

A, chondrosarcoma is a cartilage-forming tumor occurring in older adults

C, giant cell tumor contains abundant multinucleate giant cells and affects epiphyses

D, osteosarcoma is a bone-forming tumor in which malignant osteoblasts lay down osteoid

E, plasmacytoma (myeloma) is a plasma cell tumor


Question #14

A 71-year-old man has experienced aching pain in the right knee, lower back, right distal

fifth finger, and neck over the past 10 years that increases with exercise. On physical

examination, there is no joint swelling, warmth, or deformity. Some joint crepitus is

audible on moving the knee. Laboratory studies show normal levels of serum calcium,

phosphorus, alkaline phosphatase, and uric acid. Serologic testing for antibodies against

cyclic citrullinated peptides (CCP), dsDNA, and Borrelia burgdorferi is negative. What is

the most likely initiating event in this disease?

A Urate crystal deposition

B Degeneration of articular cartilage

C Synovial inflammation

D Nongonococcal urethritis

E CD4+ cell reactions against synovial antigens

Correct answer: B. Long-standing arthritis affecting multiple joints, particularly large

weight bearing joints, in an aging individual is most often due to osteoarthritis

(degenerative joint disease). For some reason, small joints of the hand and first

tarsometatarsal joint are also affected. The hallmark of this disease is degeneration of

articular cartilage.

A, urate deposition causes gout, which usually presents as a painful acute arthritis in the

toe; in most cases, serum uric acid is elevated


C, E, synovial inflammation, often caused by an autoimmune reaction involving CD4+ T

cells, is the basis of rheumatoid arthritis. This disease often presents in younger

individuals, preferentially involves small joints, and is associated with anti-CCP

antibodies.

D, some nongonococcal urethral infections such as Chlamydia are followed by a reactive

arthritis (presumably caused by a host immune response, since bacteria cannot be

detected in the inflamed joints). The arthritis is usually acute in onset and resolves

spontaneously.
Question #15

A 47-year-old woman has had increasing pain and deformities in her hands for the past 10

years. Pain and stiffness are worse in the morning and improve during the day. On

physical examination, she has metacarpophalangeal joint swelling, erythema, and

tenderness. There is a subcutaneous nodule on the ulnar aspect of the right forearm. A

biopsy specimen of the nodule has the microscopic appearance depicted in the figure.

Fig 21-38 BP10

Which of the following therapies is most likely to be effective in this patient?

A Bisphosphonates

B Broad-spectrum antibiotics

C Anti-TNF agents

D Uricosuric agents

E. Vitamin D

Correct answer: C. Arthritis affecting small joints that improves with exercise is typically

rheumatoid arthritis. The subcutaneous lesion, called a rheumatoid nodule, is unusual


but the histology (central necrosis rimmed by palisading, activated macrophages) is

classic. TNF antagonists are of great benefit in this disease; they reduce the damage and

manifestations of inflammation, although they don’t cure the disease and have to be

taken permanently.

A, bisphosphonates reduce osteoclast activity and are used in any condition characterized

by excessive bone resorption, such as osteoporosis, hypercalcemia of any cause,

metastatic bone disease, and Paget disease.

B, antibiotics are useful only in infections

D, uricosuric agents, which increase uric acid excretion in the urine, are used to treat gout.

E, Vitamin D is used in osteoporosis since it increases calcium absorption


Question #16

A 34-year-old woman has experienced malaise, fatigue, and joint pain for the past 5 months.

She has had progressive loss of joint motion, making it more difficult to use her hands.

On physical examination, the joint involvement is symmetric, and most of the affected

joints are in the hands and feet. The involved joints are swollen and warm to the touch.

The second and third digits on the right hand have a swan neck deformity, and there is

ulnar deviation of both hands. Reconstructive surgery is performed on her right hand.

Microscopic views of the excised joint capsule tissue are shown in the figure.

Fig 21-37 B and C,


BP10

Antibodies against which of the following is most likely to be present in this patient?

A DNA topoisomerase

B Calcium pyrophosphate crystals

C Borrelia burgdorferi

D Campylobacter jejuni

E Citrullinated peptides
Correct answer: E. Small joint arthritis in a young woman with “swan neck” deformity is

strongly suggestive of rheumatoid arthritis (RA). The histology shows synovitis with

chronic inflammatory infiltrates, typical of RA. Antibodies against cyclic citrullinated

peptides (CCP) are frequently seen in this disease, especially among smokers. The role

of these antibodies in disease pathogenesis is unclear. They suggest that citrullination

occurs early and citrullinated proteins in joints and other tissues may be “neoantigens”

that elicit destructive inflammatory responses.

A, antibodies against DNA topoisomerase are most often seen in systemic sclerosis

B, calcium pyrophosphate crystal deposition is seen in pseudogout and does not elicit an

antibody response

C, antibodies against Borrelia indicate prior exposure and may be seen in Lyme arthritis,

which is usually a migratory arthritis associated with involvement of skin and other

organs.

D, antibodies against Campylobacter may be seen in reactive arthritis, usually a self-

limited acute arthritis and not the progressive disease typical of RA.

20 A 55-year-old woman has had worsening problems with memory and the ability to carry

out tasks of daily living over the past year. She has had watery diarrhea for the past 3

months. Physical examination shows red, scaling skin in sun-exposed areas. Deep tendon

reflexes are normal, and sensation is intact. Which of the following diseases is she most

likely to have?
A Beriberi

B Cheilosis

C Hypothyroidism

D Marasmus

E Pellagra

Answer, E Pellagra, cause by niacin (vitamin B3) deficiency, which is characterized by

diarrhea, dermatitis, dementia, all features seen in this case

A Beriberi is a form of high output cardiac failure caused by thiamine (vitamin B1)

deficiency

B Cheilosis (angular stomatitis) is a feature of riboflavin (vitamin B2) deficiency, but

can also be seen with other vitamin deficiencies, including niacin deficiency. It is not

a specific disease.

C Hypothyroidism may present with many alterations, including skin changes

(myxedema), fatigue (due to decreased cardiac output), and constipation.

D Marasmus is a wasting condition caused by inadequate caloric intake (starvation)

21 A 52-year-old woman with a long history of ethanol abuse has had congestive heart

failure for the past year. For the past month, she has experienced increasing confusion,

disorientation, and difficulty ambulating. Physical examination shows nystagmus, ataxia

of gait, and decreased sensation in the lower extremities. Laboratory studies show

hemoglobin, 13.1 g/dL; hematocrit, 39.3%; MCV, 90 µm3; platelet count, 269,300/mm3;
and WBC count 7120/mm3. A long-term dietary deficiency of which of the following

nutrients is most likely to produce these findings?

A Folate

B Niacin

C Pyridoxine

D Riboflavin

E Thiamine

Answer, E Thiamine deficiency is marked by high output heart failure (beriberi), peripheral neuropathy, and

Wernicke encephalopathy, all of which may be present in this case.

A Folate deficiency typically presents with megaloblastic anemia and symptoms related

to cytopenias

B Niacin deficiency is characterized by diarrhea, dermatitis, dementia

C Pyridoxine deficiency may present with seborrheic dermatitis, glossitis, stomatitis,

and neuropathies.

D Riboflavin deficiency is associated with cheilosis, glossitis, seborrheic dermatitis, and

corneal abnormalities

22 A 55-year-old woman has been steadily gaining weight for the past 30 years. She

underwent a cholecystectomy for cholelithiasis 5 years ago. She does not smoke. She is

now 164 cm (5 ft 4 in) tall and weighs 126 kg (BMI 47). On physical examination, she

has decreased range of motion with pain on movement of the knees. Laboratory studies
show a serum glucose level of 176 mg/dL. This patient is at greatest risk of developing

which of the following neoplasms?

A Liposarcoma

B Endometrial carcinoma

C Hepatocellular carcinoma

D Pulmonary adenocarcinoma

E Carcinoma of the cervix

Answer, B Endometrial carcinoma. Because of her obesity, this woman is exposed to

higher levels of endogenous estrogen, which is synthesized in peripheral adipose

tissues. Sustained stimulation of the growth of uterine glands by estrogen is an

important risk factor for endometrial carcinoma.

A Liposarcoma has no known risk factors, outside of rare individuals who inherit

mutations in the TP53 tumor suppressor gene (Li-Fraumeni syndrome).

C, the major risk factors for hepatocellular carcinoma are chronic infection with

hepatitis B or C, as well as other causes of chronic liver injury.

D, Pulmonary adenocarcinoma risk is mainly associated with smoking tobacco

E Carcinoma of the cervix risk is mainly associated with chronic infection by high-risk

human papilloma viruses

23 A 45 year old female whose BMI is 35 has a consistently high caloric intake because she

lacks a feeling of satiety when eating. She is most likely to have diminished

responsiveness to which of the following molecules?


A Ghrelin

B Glucagon

C Neuropepeptide Y

D Adiponectin

E Leptin

Answer, E Leptin is a hormone released from adipocytes that acts on leptin receptors in

the hypothalamus and brain stem to suppress appetite.

A, Ghrelin is a peptide hormone released from the stomach in response to fasting that

binds receptors in the CNS and stimulates appetite.

B Glucagon is released from pancreatic alpha cells in a fashion that is negatively

regulated by glucose and insulin. It acts to raise glucose levels in the blood by

increasing gluconeogenesis by the liver.

C, Neuropeptide Y is a neurotransmitter produced in the CNS and is a potent stimulator

of food consumption via actions in the hypothalamus.

D, Adiponectin is an adipocyte-derived cytokine that acts in the liver and skeletal

muscle, thereby decreasing free fatty acid levels. There is evidence that adiponectin

levels are inappropriately low in obesity, not that tissues are unresponsive to its

effects.

24 It is 1:00 am and a hard-working second-year medical student is intent on finishing her

pathology reading assignment. Soon she begins to note that her concentration is fading

because 7 hours have passed since she had dinner, and she is feeling famished. Having
studied the chapter on ischemic heart disease, she decides to be prudent and forgoes her

favorite chocolate cookies, and instead devours two apples, gulping them down with a

glass of low-fat milk. Of the following substances, which one was most likely to have

increased rapidly when she became hungry and decreased promptly after she finished her

healthy snack?

A Leptin

B Corticotropin-releasing factor (CRF)

C Ghrelin

D Adiponectin

E Thyrotropin-releasing hormone (TRH)

Answer, C Ghrelin is a peptide produce by gastric epithelial cells that can penetrate the

CNS and directly stimulate appetite centers in the hypothalamus.

A, Leptin is a hormone produced by adipocytes that suppresses appetite

B, Corticotropin-releasing factor (CRF) is produced in the hypothalamus and stimulates

the release of adrenocorticotrophic hormone (ACTH) from the anterior pituitary.

D Adiponectin is a cytokine released from adipocytes that that acts in the liver and

skeletal muscle, thereby decreasing free fatty acid levels.

E Thyrotropin-releasing hormone (TRH) is produced in the hypothalamus and

stimulates the release of thyroid stimulating hormone (TSH) from the anterior

pituitary.
Question #13

Laboratory tests are ordered for two hospitalized patients. During the phlebotomy

procedure, the samples drawn from these patients are mislabeled. One of the patients

receives a blood transfusion later that day. Within 1 hour after the transfusion of packed

RBCs begins, the patient becomes tachycardic and hypotensive and passes pink-colored

urine. Which of the following mechanisms is most likely to be responsible for the clinical

picture described?

A Activation of leukocytes by cross linking of Fc receptors

B Antigen-antibody complex deposition in glomeruli

C Complement-mediated lysis of antibody coated red cells

D Mast cell degranulation with release of biogenic amines

E Release of tumor necrosis factor α into the circulation

Correct answer: C. The laboratory error likely led to the patient receiving a mismatched

blood transfusion. Individuals of a particular ABO blood type have “natural” antibodies

against the blood group antigens they do not express, but not against their own blood

group antigens (because their own antigens induce tolerance). The mismatched blood

probably contained antibodies against the patient’s red cells. These antibodies bound to

the red cells, activated complement, and caused massive hemolysis as well as systemic

inflammation, components of a transfusion reaction.

A, antibodies bound to antigens do activate leukocytes via Fc receptors, and may cause

antibody-dependent cellular cytotoxicity (which may kill infected and tumor cells), but

this is not the major reaction for red cell destruction


B, immune complex deposition in glomeruli would cause progressive renal disease and not

the systemic manifestations seen in this patient

D, mast cell degranulation is important in allergic reactions

E, TNF release is seen in disseminated bacterial infections and is a cause of septic shock

© Vinay Kumar

You might also like